Health Assessment Final Test Bank Questions

अब Quizwiz के साथ अपने होमवर्क और परीक्षाओं को एस करें!

When examining an infant, the nurse should examine which area first? a. Ear b. Nose c. Throat d. Abdomen

d. Abdomen Rationale: The least-distressing steps are performed first, saving the invasive steps of the examination of the eye, ear, nose, and throat until last.

CN II? What is its function? Where does it originate?

Optic Check for movement; Visual Acuity with Snellen Chart; Visual Fields with EOMs Originates in the Cerebrum

CN VII? What is its function? Where does it originate?

Incorporate CN IX, X, XII--- Raise eye brows, puff cheeks, smile, clinch teeth, stick out tongue

The external male genital structures include the: a. Testis. b. Scrotum. c. Epididymis. d. Vas deferens.

b. Scrotum. Rationale: The external male genital structures include the penis and scrotum. The testis, epididymis, and vas deferens are internal structures.

What are the four main components of the mental status assessment?

1. appearance 2. behavior 3. cognition 4. thought processes Think of the initials A, B, C, and T to help remember these categories.

CN VI? What is its function? Where does it originate?

Abducens Lateral Rectus Muscle

CN VIII? What is its function? Where does it originate?

Acoustic Test for hearing and balance

Where is the storage site of sperm?

Epididymis

CN III? What is its function? Where does it originate?

Oculomotor Responsible for all the muscless but the 4th and 6th muscles; Moving the eye lid; Pupillary respons Originates in the Brainstem

CN I? What is its function? Where does it originate?

Olfactory Checks smell Originates in the Cerebrum

CN V? What is its function? Where does it originate?

Trigeminal V1-V2-V3, Ophthalmic-Maxillary-Mandibular Blink-Sensory-Chewing

CN IV? What is its function? Where does it originate?

Trochlear; Largest nerve Enervates the superior oblique muscle Originates in the Brainstem

The nurse is reviewing a patients medical record and notes that he is in a coma. Using the Glasgow Coma Scale, which number indicates that the patient is in a coma? a. 6 b. 12 c. 15 d. 24

a. 6 Rationale: A fully alert, normal person has a score of 15, whereas a score of 7 or less reflects coma on the Glasgow Coma Scale (see Figure 23-59).

The nurse is assessing the mental status of a child. Which statement about children and mental status is true? a. All aspects of mental status in children are interdependent. b. Children are highly labile and unstable until the age of 2 years. c. Children's mental status is largely a function of their parents level of functioning until the age of 7 years. d. A child's mental status is impossible to assess until the child develops the ability to concentrate.

a. All aspects of mental status in children are interdependent. Rationale: Separating and tracing the development of only one aspect of mental status is difficult. All aspects are interdependent. For example, consciousness is rudimentary at birth because the cerebral cortex is not yet developed. The infant cannot distinguish the self from the mothers body. The other statements are not true.

A 75-year-old woman is at the office for a preoperative interview. The nurse is aware that the interview may take longer than interviews with younger persons. What is the reason for this? a. An aged person has a longer story to tell. b. An aged person is usually lonely and likes to have someone with whom to talk. c. Aged persons lose much of their mental abilities and require longer time to complete an interview. d. As a person ages, he or she is unable to hear; thus the interviewer usually needs to repeat much of what is said.

a. An aged person has a longer story to tell. Rationale: The interview usually takes longer with older adults because they have a longer story to tell. It is not necessarily true that all older adults are lonely, have lost mental abilities, or are hard of hearing.

The uterus is usually positioned tilting forward and superior to the bladder. This position is known as: a. Anteverted and anteflexed. b. Retroverted and anteflexed. c. Retroverted and retroflexed. d. Superiorverted and anteflexed.

a. Anteverted and anteflexed. Rationale: The uterus is freely movable, not fixed, and usually tilts forward and superior to the bladder (a position labeled as anteverted and anteflexed).

A 6-month-old infant has been brought to the well-child clinic for a check-up. She is currently sleeping. What should the nurse do first when beginning the examination? a. Auscultate the lungs and heart while the infant is still sleeping. b. Examine the infants hips, because this procedure is uncomfortable. c. Begin with the assessment of the eye, and continue with the remainder of the examination in a head-to-toe approach. d. Wake the infant before beginning any portion of the examination to obtain the most accurate assessment of body systems.

a. Auscultate the lungs and heart while the infant is still sleeping. Rationale: When the infant is quiet or sleeping is an ideal time to assess the cardiac, respiratory, and abdominal systems. Assessment of the eye, ear, nose, and throat are invasive procedures that should be performed at the end of the examination.

A 43-year-old woman is at the clinic for a routine examination. She reports that she has had a breast lump in her right breast for years. Recently, it has begun to change in consistency and is becoming harder. She reports that 5 years ago her physician evaluated the lump and determined that it was nothing to worry about. The examination validates the presence of a mass in the right upper outer quadrant at 1 o'clock, approximately 5 cm from the nipple. It is firm, mobile, and non-tender, with borders that are not well defined. The nurse replies: a. Because of the change in consistency of the lump, it should be further evaluated by a physician. b. The changes could be related to your menstrual cycles. Keep track of the changes in the mass each month. c. The lump is probably nothing to worry about because it has been present for years and was determined to be noncancerous 5 years ago. d. Because you are experiencing no pain and the size has not changed, you should continue to monitor the lump and return to the clinic in 3 months.

a. Because of the change in consistency of the lump, it should be further evaluated by a physician. Rationale: A lump that has been present for years and is not exhibiting changes may not be serious but should still be explored. Any recent change or a new lump should be evaluated. The other responses are not correct.

A 55-year-old man is in the clinic for a yearly checkup. He is worried because his father died of prostate cancer. The nurse knows which tests should be performed at this time? Select all that apply. a. Blood test for prostate-specific antigen (PSA) b. Urinalysis c. Transrectal ultrasound d. Digital rectal examination (DRE) e. Prostate biopsy

a. Blood test for prostate-specific antigen (PSA) d. Digital rectal examination (DRE) Rationale: Prostate cancer is typically detected by testing the blood for PSA or by a DRE. It is recommended that both PSA and DRE be offered to men annually, beginning at age 50 years. If the PSA is elevated, then further laboratory work or a transrectal ultrasound (TRUS) and biopsy may be recommended.

When performing a genitourinary assessment, the nurse notices that the urethral meatus is ventrally positioned. This finding is: a. Called hypospadias. b. A result of phimosis. c. Probably due to a stricture. d. Often associated with aging.

a. Called hypospadias. Rationale: Normally, the urethral meatus is positioned just about centrally. Hypospadias is the ventral location of the urethral meatus. The position of the meatus does not change with aging. Phimosis is the inability to retract the foreskin. A stricture is a narrow opening of the meatus.

The nurse is conducting an interview in an outpatient clinic and is using a computer to record data. Which are the best uses of the computer in this situation? Select all that apply. a. Collect the patients data in a direct, face-to-face manner. b. Enter all the data as the patient states them. c. Ask the patient to wait as the nurse enters the data. d. Type the data into the computer after the narrative is fully explored. e. Allow the patient to see the monitor during typing.

a. Collect the patients data in a direct, face-to-face manner. d. Type the data into the computer after the narrative is fully explored. e. Allow the patient to see the monitor during typing. Rationale: The use of a computer can become a barrier. The nurse should begin the interview as usual by greeting the patient, establishing rapport, and collecting the patients narrative story in a direct, face-to-face manner. Only after the narrative is fully explored should the nurse type data into the computer. When typing, the nurse should position the monitor so that the patient can see it.

The patients record, laboratory studies, objective data, and subjective data combine to form the: a. Data base b. Admitting data c. Financial statement d. Discharge summary

a. Data base Rationale: Together with the patients record and laboratory studies, the objective and subjective data form the data base. The other items are not part of the patients record, laboratory studies, or data.

The nurse is interviewing a male patient who has a hearing impairment. What techniques would be most beneficial in communicating with this patient? a. Determine the communication method he prefers. b. Avoid using facial and hand gestures because most hearing-impaired people find this degrading. c. Request a sign language interpreter before meeting with him to help facilitate the communication. d. Speak loudly and with exaggerated facial movement when talking with him because doing so will help him lip read.

a. Determine the communication method he prefers. Rationale: The nurse should ask the deaf person the preferred way to communicate by signing, lip reading, or writing. If the person prefers lip reading, then the nurse should be sure to face him squarely and have good lighting on the nurses face. The nurse should not exaggerate lip movements because this distorts words. Similarly, shouting distorts the reception of a hearing aid the person may wear. The nurse should speak slowly and supplement his or her voice with appropriate hand gestures or pantomime.

The nurse is assessing a patient who is admitted with possible delirium. Which of these are manifestations of delirium? Select all that apply. a. Develops over a short period. b. Person is experiencing apraxia. c. Person is exhibiting memory impairment or deficits. d. Occurs as a result of a medical condition, such as systemic infection. e. Person is experiencing agnosia.

a. Develops over a short period. c. Person is exhibiting memory impairment or deficits. d. Occurs as a result of a medical condition, such as systemic infection. Rationale: Delirium is a disturbance of consciousness that develops over a short period and may be attributable to a medical condition. Memory deficits may also occur. Apraxia and agnosia occur with dementia.

Which of these statements is most appropriate when the nurse is obtaining a genitourinary history from an older man? a. Do you need to get up at night to urinate? b. Do you experience nocturnal emissions, or wet dreams? c. Do you know how to perform a testicular self-examination? d. Has anyone ever touched your genitals when you did not want them to?

a. Do you need to get up at night to urinate? Rationale: The older male patient should be asked about the presence of nocturia. Awaking at night to urinate may be attributable to a diuretic medication, fluid retention from mild heart failure or varicose veins, or fluid ingestion 3 hours before bedtime, especially coffee and alcohol. The other questions are more appropriate for younger men.

A 59-year-old patient has been diagnosed with prostatitis and is being seen at the clinic for complaints of burning and pain during urination. He is experiencing: a. Dysuria. b. Nocturia. c. Polyuria. d. Hematuria.

a. Dysuria. Rationale: Dysuria (burning with urination) is common with acute cystitis, prostatitis, and urethritis. Nocturia is voiding during the night. Polyuria is voiding in excessive quantities. Hematuria is voiding with blood in the urine.

A patient will be ready to be discharged from the hospital soon, and the patients family members are concerned about whether the patient is able to walk safely outside alone. The nurse will perform which test to assess this? a. Get Up and Go Test b. Performance ADLs c. Physical Performance Test d. Tinetti Gait and Balance Evaluation

a. Get Up and Go Test Rationale: The Get Up and Go Test is a reliable and valid test to quantify functional mobility. The test is quick, requires little training and no special equipment, and is appropriate to use in many settings including hospitals and clinics. This instrument has been shown to predict a persons ability to go safely outside alone. The Performance of ADLs test has a trained observer actually observing as a patient performs various ADLs. The Physical Performance Test assesses upper body fine motor and coarse motor activities, as well as balance, mobility, coordination, and endurance. The Tinetti Gait and Balance Evaluation assesses gait and balance and provides information about fall risk.

The nurse discovers speech problems in a patient during an assessment. The patient has spontaneous speech, but it is mostly absent or is reduced to a few stereotypical words or sounds. This finding reflects which type of aphasia? a. Global b. Brocas c. Dysphonic d. Wernicke's

a. Global Rationale: Global aphasia is the most common and severe form of aphasia. Spontaneous speech is absent or reduced to a few stereotyped words or sounds, and prognosis for language recovery is poor. (Brocas aphasia and Wernickes aphasia are described in Table 5-4.) Dysphonic aphasia is not a valid condition.

A patients vision is recorded as 20/80 in each eye. The nurse interprets this finding to mean that the patient: a. Has poor vision. b. Has acute vision. c. Has normal vision. d. Is presbyopic.

a. Has poor vision. Rationale: Normal visual acuity is 20/20 in each eye; the larger the denominator, the poorer the vision.

A patient visits the clinic to ask about smoking cessation. He has smoked heavily for 30 years and wants to stop cold turkey. He asks the nurse, What symptoms can I expect if I do this? Which of these symptoms should the nurse share with the patient as possible symptoms of nicotine withdrawal? Select all that apply. a. Headaches b. Hunger c. Sleepiness d. Restlessness e. Nervousness f. Sweating

a. Headaches b. Hunger d. Restlessness e. Nervousness Rationale: Symptoms of nicotine withdrawal include vasodilation, headaches, anger, irritability, frustration, anxiety, nervousness, awakening at night, difficulty concentrating, depression, hunger, impatience, and the desire to smoke (see Table 6-7).

The nurse is assessing a patient who has been admitted for cirrhosis of the liver, secondary to chronic alcohol use. During the physical assessment, the nurse looks for cardiac problems that are associated with chronic use of alcohol, such as: a. Hypertension. b. Ventricular fibrillation. c. Bradycardia. d. Mitral valve prolapse.

a. Hypertension. Rationale: Even moderate drinking leads to hypertension and cardiomyopathy, with an increase in left ventricular mass, dilation of ventricles, and wall thinning. Ventricular fibrillation, bradycardia, and mitral valve prolapse are not associated with chronic heavy use of alcohol.

The nurse is reviewing data collected after an assessment. Of the data listed below, which would be considered related cues that would be clustered together during data analysis? Select all that apply. a. Inspiratory wheezes noted in left lower lobes b. Hypoactive bowel sounds c. Nonproductive cough d. Edema, +2, noted on left hand e. Patient reports dyspnea upon exertion f. Rate of respirations 16 breaths per minute

a. Inspiratory wheezes noted in left lower lobes c. Nonproductive cough e. Patient reports dyspnea upon exertion f. Rate of respirations 16 breaths per minute Rationale: Clustering related cues help the nurse recognize relationships among the data. The cues related to the patients respiratory status (e.g., wheezes, cough, report of dyspnea, respiration rate and rhythm) are all related. Cues related to bowels and peripheral edema are not related to the respiratory cues.

A patient drifts off to sleep when she is not being stimulated. The nurse can easily arouse her by calling her name, but the patient remains drowsy during the conversation. The best description of this patients level of consciousness would be: a. Lethargic b. Obtunded c. Stuporous d. Semialert

a. Lethargic Rationale: Lethargic (or somnolent) is when the person is not fully alert, drifts off to sleep when not stimulated, and can be aroused when called by name in a normal voice but looks drowsy. He or she appropriately responds to questions or commands, but thinking seems slow and fuzzy. He or she is inattentive and loses the train of thought. Spontaneous movements are decreased. (See Table 5-3 for the definitions of the other terms.)

A 68-year-old woman is in the eye clinic for a checkup. She tells the nurse that she has been having trouble reading the paper, sewing, and even seeing the faces of her grandchildren. On examination, the nurse notes that she has some loss of central vision but her peripheral vision is normal. These findings suggest that she may have: a. Macular degeneration. b. Vision that is normal for someone her age. c. The beginning stages of cataract formation. d. Increased intraocular pressure or glaucoma.

a. Macular degeneration. Rationale: Macular degeneration is the most common cause of blindness. It is characterized by the loss of central vision. Cataracts would show lens opacity. Chronic open-angle glaucoma, the most common type of glaucoma, involves a gradual loss of peripheral vision. These findings are not consistent with vision that is considered normal at any age.

When performing an otoscopic examination of a 5-year-old child with a history of chronic ear infections, the nurse sees that his right tympanic membrane is amber-yellow in color and that air bubbles are visible behind the tympanic membrane. The child reports occasional hearing loss and a popping sound with swallowing. The preliminary analysis based on this information is that the child: a. Most likely has serous otitis media. b. Has an acute purulent otitis media. c. Has evidence of a resolving cholesteatoma. d. Is experiencing the early stages of perforation.

a. Most likely has serous otitis media. Rationale: An amber-yellow color to the tympanic membrane suggests serum or pus in the middle ear. Air or fluid or bubbles behind the tympanic membrane are often visible. The patient may have feelings of fullness, transient hearing loss, and a popping sound with swallowing. These findings most likely suggest that the child has serous otitis media. The other responses are not correct.

The nurse is assessing the breasts of a 68-year-old woman and discovers a mass in the upper outer quadrant of the left breast. When assessing this mass, the nurse is aware that characteristics of a cancerous mass include which of the following? Select all that apply. a. Nontender mass b. Dull, heavy pain on palpation c. Rubbery texture and mobile d. Hard, dense, and immobile e. Regular border f. Irregular, poorly delineated border

a. Nontender mass d. Hard, dense, and immobile f. Irregular, poorly delineated border Rationale: Cancerous breast masses are solitary, unilateral, and nontender. They are solid, hard, dense, and fixed to underlying tissues or skin as cancer becomes invasive. Their borders are irregular and poorly delineated. They are often painless, although the person may experience pain. They are most common in the upper outer quadrant. A dull, heavy pain on palpation and a mass with a rubbery texture and a regular border are characteristics of benign breast disease.

Barriers to incorporating EBP include: a. Nurses lack of research skills in evaluating the quality of research studies b. Lack of significant research studies c. Insufficient clinical skills of nurses d. Inadequate physical assessment skills

a. Nurses lack of research skills in evaluating the quality of research studies Rationale: As individuals, nurses lack research skills in evaluating the quality of research studies, are isolated from other colleagues who are knowledgeable in research, and often lack the time to visit the library to read research. The other responses are not considered barriers.

After completing an initial assessment of a patient, the nurse has charted that his respirations are eupneic and his pulse is 58 beats per minute. These types of data would be: a. Objective b. Reflective c. Subjective d. Introspective

a. Objective Rationale: Objective data are what the health professional observes by inspecting, percussing, palpating, and auscultating during the physical examination. Subjective data is what the person says about him or herself during history taking. The terms reflective and introspective are not used to describe data.

During an examination, the nurse notices that the patient stumbles a little while walking, and, when she sits down, she holds on to the sides of the chair. The patient states, It feels like the room is spinning! The nurse notices that the patient is experiencing: a. Objective vertigo. b. Subjective vertigo. c. Tinnitus. d. Dizziness.

a. Objective vertigo. Rationale: With objective vertigo, the patient feels like the room spins; with subjective vertigo, the person feels like he or she is spinning. Tinnitus is a sound that comes from within a person; it can be a ringing, crackling, or buzzing sound. It accompanies some hearing or ear disorders. Dizziness is not the same as true vertigo; the person who is dizzy may feel unsteady and lightheaded.

The nurse is preparing to perform a functional assessment of an older patient and knows that a good approach would be to: a. Observe the patients ability to perform the tasks. b. Ask the patients wife how he does when performing tasks. c. Review the medical record for information on the patients abilities. d. Ask the patients physician for information on the patients abilities.

a. Observe the patients ability to perform the tasks. Rationale: Two approaches are used to perform a functional assessment: (1) asking individuals about their ability to perform the tasks (self-reports), or (2) actually observing their ability to perform the tasks. For persons with memory problems, the use of surrogate reporters (proxy reports), such as family members or caregivers, may be necessary, keeping in mind that they may either overestimate or underestimate the persons actual abilities.

The nurse is performing a general survey. Which action is a component of the general survey? a. Observing the patients body stature and nutritional status b. Interpreting the subjective information the patient has reported c. Measuring the patients temperature, pulse, respirations, and blood pressure d. Observing specific body systems while performing the physical assessment

a. Observing the patients body stature and nutritional status Rationale: The general survey is a study of the whole person that includes observing the patients physical appearance, body structure, mobility, and behavior.

The nurse is reviewing percussion techniques with a newly graduated nurse. Which technique, if used by the new nurse, indicates that more review is needed? a. Percussing once over each area b. Quickly lifting the striking finger after each stroke c. Striking with the fingertip, not the finger pad d. Using the wrist to make the strikes, not the arm

a. Percussing once over each area Rationale: For percussion, the nurse should percuss two times over each location. The striking finger should be quickly lifted because a resting finger damps off vibrations. The tip of the striking finger should make contact, not the pad of the finger. The wrist must be relaxed and is used to make the strikes, not the arm.

The nurse is preparing for a certification course in skin care and needs to be familiar with the various lesions that may be identified on assessment of the skin. Which of the following definitions are correct? Select all that apply. a. Petechiae: Tiny punctate hemorrhages, 1 to 3 mm, round and discrete, dark red, purple, or brown in color b. Bulla: Elevated, circumscribed lesion filled with turbid fluid (pus) c. Papule: Hypertrophic scar d. Vesicle: Known as a friction blister e. Nodule: Solid, elevated, and hard or soft growth that is larger than 1 cm

a. Petechiae: Tiny punctate hemorrhages, 1 to 3 mm, round and discrete, dark red, purple, or brown in color d. Vesicle: Known as a friction blister e. Nodule: Solid, elevated, and hard or soft growth that is larger than 1 cm Rationale: A pustule is an elevated, circumscribed lesion filled with turbid fluid (pus). A hypertrophic scar is a keloid. A bulla is larger than 1 cm and contains clear fluid. A papule is solid and elevated but measures less than 1 cm.

During a physical examination, the nurse finds that a male patients foreskin is fixed and tight and will not retract over the glans. The nurse recognizes that this condition is: a. Phimosis. b. Epispadias. c. Urethral stricture. d. Peyronie disease.

a. Phimosis. Rationale: With phimosis, the foreskin is nonretractable, forming a pointy tip of the penis with a tiny orifice at the end of the glans. The foreskin is advanced and so tight that it is impossible to retract over the glans. This condition may be congenital or acquired from adhesions related to infection. (See Table 24-3 for information on urethral stricture. See Table 24-4 for information on epispadias and Peyronie disease.)

A 16-year-old boy is brought to the clinic for a problem that he refused to let his mother see. The nurse examines him, and finds that he has scrotal swelling on the left side. He had the mumps the previous week, and the nurse suspects that he has orchitis. Which of the following assessment findings support this diagnosis? Select all that apply. a. Swollen testis b. Mass that transilluminates c. Mass that does not transilluminate d. Scrotum that is nontender upon palpation e. Scrotum that is tender upon palpation f. Scrotal skin that is reddened

a. Swollen testis c. Mass that does not transilluminate e. Scrotum that is tender upon palpation f. Scrotal skin that is reddened Rationale: With orchitis, the testis is swollen, with a feeling of weight, and is tender or painful. The mass does not transilluminate, and the scrotal skin is reddened. Transillumination of a mass occurs with a hydrocele, not orchitis.

A patient is admitted to the unit after an automobile accident. The nurse begins the mental status examination and finds that the patient has dysarthric speech and is lethargic. The nurses best approach regarding this examination is to: a. Plan to defer the rest of the mental status examination. b. Skip the language portion of the examination, and proceed onto assessing mood and affect. c. Conduct an in-depth speech evaluation, and defer the mental status examination to another time. d. Proceed with the examination, and assess the patient for suicidal thoughts because dysarthria is often accompanied by severe depression.

a. Plan to defer the rest of the mental status examination. Rationale: In the mental status examination, the sequence of steps forms a hierarchy in which the most basic functions (consciousness, language) are assessed first. The first steps must be accurately assessed to ensure validity of the steps that follow. For example, if consciousness is clouded, then the person cannot be expected to have full attention and to cooperate with new learning. If language is impaired, then a subsequent assessment of new learning or abstract reasoning (anything that requires language functioning) can give erroneous conclusions.

When performing a musculoskeletal assessment, the nurse knows that the correct approach for the examination should be: a. Proximal to distal. b. Distal to proximal. c. Posterior to anterior. d. Anterior to posterior.

a. Proximal to distal. Rationale: The musculoskeletal assessment should be performed in an orderly approach, head to toe, proximal to distal, from the midline outward. The other options are not correct.

The nurse is preparing to do an otoscopic examination on a 2-year-old child. Which one of these reflects the correct procedure? a. Pulling the pinna down b. Pulling the pinna up and back c. Slightly tilting the child's head toward the examiner d. Instructing the child to touch his chin to his chest

a. Pulling the pinna down Rationale: For an otoscopic examination on an infant or on a child under 3 years of age, the pinna is pulled down. The other responses are not part of the correct procedure.

The nurse is testing a patients visual accommodation, which refers to which action? a. Pupillary constriction when looking at a near object b. Pupillary dilation when looking at a far object c. Changes in peripheral vision in response to light d. Involuntary blinking in the presence of bright light

a. Pupillary constriction when looking at a near object Rationale: The muscle fibers of the iris contract the pupil in bright light and accommodate for near vision, which also results in pupil constriction. The other responses are not correct.

The nurse knows that one advantage of the tympanic membrane thermometer (TMT) is that: a. Rapid measurement is useful for uncooperative younger children. b. Using the TMT is the most accurate method for measuring body temperature in newborn infants. c. Measuring temperature using the TMT is inexpensive. d. Studies strongly support the use of the TMT in children under the age 6 years.

a. Rapid measurement is useful for uncooperative younger children. Rationale: The TMT is useful for young children who may not cooperate for oral temperatures and fear rectal temperatures. However, the use a TMT with newborn infants and young children is conflicting.

The nurse is assessing an older adults advanced activities of daily living (AADLs), which would include: a. Recreational activities. b. Meal preparation. c. Balancing the checkbook. d. Self-grooming activities.

a. Recreational activities. Rationale: AADLs are activities that an older adult performs such as occupational and recreational activities. Self-grooming activities are basic ADLs; meal preparation and balancing the checkbook are considered IADLs.

While obtaining a health history of a 3-month-old infant from the mother, the nurse asks about the infants ability to suck and grasp the mothers finger. What is the nurse assessing? a. Reflexes b. Intelligence c. CNs d. Cerebral cortex function

a. Reflexes Rationale: Questions regarding reflexes include such questions as, What have you noticed about the infants behavior, Are the infants sucking and swallowing seem coordinated, and Does the infant grasp your finger? The other responses are incorrect.

A 21-year-old patient has a head injury resulting from trauma and is unconscious. There are no other injuries. During the assessment what would the nurse expect to find when testing the patients deep tendon reflexes? a. Reflexes will be normal. b. Reflexes cannot be elicited. c. All reflexes will be diminished but present. d. Some reflexes will be present, depending on the area of injury.

a. Reflexes will be normal. Rationale: A reflex is a defense mechanism of the nervous system. It operates below the level of conscious control and permits a quick reaction to potentially painful or damaging situations.

The nurse is assessing the vital signs of a 3-year-old patient who appears to have an irregular respiratory pattern. How should the nurse assess this childs respirations? a. Respirations should be counted for 1 full minute, noticing rate and rhythm. b. Child's pulse and respirations should be simultaneously checked for 30 seconds. c. Child's respirations should be checked for a minimum of 5 minutes to identify any variations in his or her respiratory pattern. d. Patients respirations should be counted for 15 seconds and then multiplied by 4 to obtain the number of respirations per minute.

a. Respirations should be counted for 1 full minute, noticing rate and rhythm. Rationale: Respirations are counted for 1 full minute if an abnormality is suspected. The other responses are not correct actions.

When examining a patients CN function, the nurse remembers that the muscles in the neck that are innervated by CN XI are the: a. Sternomastoid and trapezius. b. Spinal accessory and omohyoid. c. Trapezius and sternomandibular. d. Sternomandibular and spinal accessory.

a. Sternomastoid and trapezius. Rationale: The major neck muscles are the sternomastoid and the trapezius. They are innervated by CN XI, the spinal accessory.

A 14-year-old girl is anxious about not having reached menarche. When taking the health history, the nurse should ascertain which of the following? The age that: a. The girl began to develop breasts. b. Her mother developed breasts. c. She began to develop pubic hair. d. She began to develop axillary hair.

a. The girl began to develop breasts. Rationale: Full development from stage 2 to stage 5 takes an average of 3 years, although the range is 1 to 6 years. Pubic hair develops during this time, and axillary hair appears 2 years after the onset of pubic hair. The beginning of breast development precedes menarche by approximately 2 years. Menarche occurs in breast development stage 3 or 4, usually just after the peak of the adolescent growth spurt, which occurs around age 12 years (see Figure 17-6).

The nurse is performing an external eye examination. Which statement regarding the outer layer of the eye is true? a. The outer layer of the eye is very sensitive to touch. b. The outer layer of the eye is darkly pigmented to prevent light from reflecting internally. c. The trigeminal nerve (CN V) and the trochlear nerve (CN IV) are stimulated when the outer surface of the eye is stimulated. d. The visual receptive layer of the eye in which light waves are changed into nerve impulses is located in the outer layer of the eye.

a. The outer layer of the eye is very sensitive to touch. Rationale: The cornea and the sclera make up the outer layer of the eye. The cornea is very sensitive to touch. The middle layer, the choroid, has dark pigmentation to prevent light from reflecting internally. The trigeminal nerve (CN V) and the facial nerve (CN VII) are stimulated when the outer surface of the eye is stimulated. The retina, in the inner layer of the eye, is where light waves are changed into nerve impulses.

While measuring a patients blood pressure, the nurse uses the proper technique to obtain an accurate reading. Which of these situations will result in a falsely high blood pressure reading? Select all that apply. a. The person supports his or her own arm during the blood pressure reading. b. The blood pressure cuff is too narrow for the extremity. c. The arm is held above level of the heart. d. The cuff is loosely wrapped around the arm. e. The person is sitting with his or her legs crossed. f. The nurse does not inflate the cuff high enough.

a. The person supports his or her own arm during the blood pressure reading. b. The blood pressure cuff is too narrow for the extremity. d. The cuff is loosely wrapped around the arm. e. The person is sitting with his or her legs crossed. Rationale: Several factors can result in blood pressure readings that are too high or too low. Having the patients arm held above the level of the heart is one part of the correct technique. (Refer to Table 9-5, Common Errors in Blood Pressure Measurement.)

During a follow-up visit, the nurse discovers that a patient has not been taking his insulin on a regular basis. The nurse asks, Why haven't you taken your insulin? Which statement is an appropriate evaluation of this question? a. This question may place the patient on the defensive. b. This question is an innocent search for information. c. Discussing his behavior with his wife would have been better. d. A direct question is the best way to discover the reasons for his behavior.

a. This question may place the patient on the defensive. Rationale: The adults use of why questions usually implies blame and condemnation and places the person on the defensive. The other statements are not correct.

During an examination, the nurse observes a female patients vestibule and expects to see the: a. Urethral meatus and vaginal orifice. b. Vaginal orifice and vestibular (Bartholin) glands. c. Urethral meatus and paraurethral (Skene) glands. d. Paraurethral (Skene) and vestibular (Bartholin) glands.

a. Urethral meatus and vaginal orifice. Rationale: The labial structures encircle a boat-shaped space, or cleft, termed the vestibule. Within the vestibule are numerous openings. The urethral meatus and vaginal orifice are visible. The ducts of the paraurethral (Skene) glands and the vestibular (Bartholin) glands are present but not visible.

The nurse is preparing to palpate the thorax and abdomen of a patient. Which of these statements describes the correct technique for this procedure? Select all that apply. a. Warm the hands first before touching the patient. b. For deep palpation, use one long continuous palpation when assessing the liver. c. Start with light palpation to detect surface characteristics. d. Use the fingertips to examine skin texture, swelling, pulsation, and presence of lumps. e. Identify any tender areas, and palpate them last. f. Use the palms of the hands to assess temperature of the skin.

a. Warm the hands first before touching the patient. c. Start with light palpation to detect surface characteristics. d. Use the fingertips to examine skin texture, swelling, pulsation, and presence of lumps. e. Identify any tender areas, and palpate them last. Rationale: The hands should always be warmed before beginning palpation. Intermittent pressure rather than one long continuous palpation is used; any tender areas are identified and palpated last. Fingertips are used to examine skin texture, swelling, pulsation, and the presence of lumps. The dorsa (backs) of the hands are used to assess skin temperature because the skin on the dorsa is thinner than on the palms.

The nurse is assessing a patients headache pain. Which questions reflect one or more of the critical characteristics of symptoms that should be assessed? Select all that apply. a. Where is the headache pain? b. Did you have these headaches as a child? c. On a scale of 1 to 10, how bad is the pain? d. How often do the headaches occur? e. What makes the headaches feel better? f. Do you have any family history of headaches?

a. Where is the headache pain? c. On a scale of 1 to 10, how bad is the pain? d. How often do the headaches occur? e. What makes the headaches feel better? Rationale: The mnemonic PQRSTU may help the nurse remember to address the critical characteristics that need to be assessed: (1) P: provocative or palliative; (2) Q: quality or quantity; (3) R: region or radiation; (4) S: severity scale; (5) T: timing; and (6) U: understand the patients perception. Asking, Where is the pain? reflects region. Asking the patient to rate the pain on a 1 to 10 scale reflects severity. Asking How often reflects timing. Asking what makes the pain better reflects provocative. The other options reflect health history and family history.

The nurse is assisting with a BSE clinic. Which of these women reflect abnormal findings during the inspection phase of breast examination? a. Woman whose nipples are in different planes (deviated). b. Woman whose left breast is slightly larger than her right. c. Non-pregnant woman whose skin is marked with linear striae. d. Pregnant woman whose breasts have a fine blue network of veins visible under the skin.

a. Woman whose nipples are in different planes (deviated). Rationale: The nipples should be symmetrically placed on the same plane on the two breasts. With deviation in pointing, an underlying cancer may cause fibrosis in the mammary ducts, which pulls the nipple angle toward it. The other examples are normal findings (see Table 17-3).

When the nurse is assessing the deep tendon reflexes (DTRs) on a woman who is 32 weeks pregnant, which of these would be considered a normal finding on a 0 to 4+ scale? a. Absent DTRs b. 2+ c. 4+ d. Brisk reflexes and the presence of clonus

b. 2+ Rationale: Normally during pregnancy, the DTRs are 1+ to 2+ and bilaterally equal. Brisk or greater than 2+ DTRs and the presence of clonus are abnormal and may be associated with an elevated blood pressure and cerebral edema in the preeclamptic woman.

The nurse has collected the following information on a patient: palpated blood pressure 180 mm Hg; auscultated blood pressure 170/100 mm Hg; apical pulse 60 beats per minute; radial pulse 70 beats per minute. What is the patients pulse pressure? a. 10 b. 70 c. 80 d. 100

b. 70 Rationale: Pulse pressure is the difference between systolic and diastolic blood pressure (170-100 = 70) and reflects the stroke volume.

During an examination of a 7-year-old girl, the nurse notices that the girl is showing breast budding. What should the nurse do next? a. Ask the young girl if her periods have started. b. Assess the girls weight and body mass index (BMI). c. Ask the girls mother at what age she started to develop breasts. d. Nothing; breast budding is a normal finding.

b. Assess the girls weight and body mass index (BMI). Rationale: Research has shown that girls with overweight or obese BMI levels have a higher occurrence of early onset of breast budding (before age 8 years for black girls and age 10 years for white girls) and early menarche.

The nurse is assessing the joints of a woman who has stated, I have a long family history of arthritis, and my joints hurt. The nurse suspects that she has osteoarthritis. Which of these are symptoms of osteoarthritis? Select all that apply. a. Symmetric joint involvement b. Asymmetric joint involvement c. Pain with motion of affected joints d. Affected joints are swollen with hard, bony protuberances e. Affected joints may have heat, redness, and swelling

b. Asymmetric joint involvement c. Pain with motion of affected joints d. Affected joints are swollen with hard, bony protuberances Rationale: In osteoarthritis, asymmetric joint involvement commonly affects hands, knees, hips, and lumbar and cervical segments of the spine. Affected joints have stiffness, swelling with hard bony protuberances, pain with motion, and limitation of motion. The other options reflect the signs of rheumatoid arthritis.

A 52-year-old woman has a papule on her nose that has rounded, pearly borders and a central red ulcer. She said she first noticed it several months ago and that it has slowly grown larger. The nurse suspects which condition? a. Acne b. Basal cell carcinoma c. Melanoma d. Squamous cell carcinoma

b. Basal cell carcinoma Rationale: Basal cell carcinoma usually starts as a skin-colored papule that develops rounded, pearly borders with a central red ulcer. It is the most common form of skin cancer and grows slowly. This description does not fit acne lesions. (See Table 12-11 for descriptions of melanoma and squamous cell carcinoma.)

The nurse is assessing a 16-year-old patient who has suffered head injuries from a recent motor vehicle accident. Which of these statements indicates the most important reason for assessing for any drainage from the ear canal? a. If the drum has ruptured, then purulent drainage will result. b. Bloody or clear watery drainage can indicate a basal skull fracture. c. The auditory canal many be occluded from increased cerumen. d. Foreign bodies from the accident may cause occlusion of the canal.

b. Bloody or clear watery drainage can indicate a basal skull fracture. Rationale: Frank blood or clear watery drainage (cerebrospinal leak) after a trauma suggests a basal skull fracture and warrants immediate referral. Purulent drainage indicates otitis externa or otitis media.

When examining the face of a woman who is 28 weeks pregnant, the nurse notices the presence of a butterfly-shaped increase in pigmentation on the face. The proper term for this finding in the documentation is: a. Striae. b. Chloasma. c. Linea nigra. d. Mask of pregnancy.

b. Chloasma. Rationale: Chloasma is a butterfly-shaped increase in pigmentation on the face. It is known as the mask of pregnancy, but when documenting, the nurse should use the correct medical term, chloasma. Striae is the term for stretch marks. The linea nigra is a hyperpigmented line that begins at the sternal notch and extends down the abdomen through the umbilicus to the pubis.

A patient reports excruciating headache pain on one side of his head, especially around his eye, forehead, and cheek that has lasted approximately to 2 hours, occurring once or twice each day. The nurse should suspect: a. Hypertension. b. Cluster headaches. c. Tension headaches. d. Migraine headaches.

b. Cluster headaches. Rationale: Cluster headaches produce pain around the eye, temple, forehead, and cheek and are unilateral and always on the same side of the head. They are excruciating and occur once or twice per day and last to 2 hours each.

A patient with a known history of heavy alcohol use has been admitted to the ICU after he was found unconscious outside a bar. The nurse closely monitors him for symptoms of withdrawal. Which of these symptoms may occur during this time? Select all that apply. a. Bradycardia b. Coarse tremor of the hands c. Transient hallucinations d. Somnolence e. Sweating

b. Coarse tremor of the hands c. Transient hallucinations e. Sweating Rationale: Symptoms of uncomplicated alcohol withdrawal start shortly after the cessation of drinking, peak at the second day, and improve by the fourth or fifth day. Symptoms include coarse tremors of the hands, tongue, and eyelids; anorexia; nausea and vomiting; autonomic hyperactivity (e.g., tachycardia, sweating, elevated blood pressure); and transient hallucinations, among other symptoms (see Table 6-7).

The ability that humans have to perform very skilled movements such as writing is controlled by the: a. Basal ganglia. b. Corticospinal tract. c. Spinothalamic tract. d. Extrapyramidal tract.

b. Corticospinal tract. Rationale: Corticospinal fibers mediate voluntary movement, particularly very skilled, discrete, and purposeful movements, such as writing. The corticospinal tract, also known as the pyramidal tract, is a newer, higher motor system that humans have that permits very skilled and purposeful movements. The other responses are not related to skilled movements.

A patient is unable to differentiate between sharp and dull stimulation to both sides of her face. The nurse suspects: a. Bell palsy. b. Damage to the trigeminal nerve. c. Frostbite with resultant paresthesia to the cheeks. d. Scleroderma.

b. Damage to the trigeminal nerve. Rationale: Facial sensations of pain or touch are mediated by CN V, which is the trigeminal nerve. Bell palsy is associated with CN VII damage. Frostbite and scleroderma are not associated with this problem.

Which of these assessment findings would the nurse expect to see when examining the eyes of a black patient? a. Increased night vision b. Dark retinal background c. Increased photosensitivity d. Narrowed palpebral fissures

b. Dark retinal background Rationale: An ethnically based variability in the color of the iris and in retinal pigmentation exists, with darker irides having darker retinas behind them.

The nurse will perform a palpated pressure before auscultating blood pressure. The reason for this is to: a. More clearly hear the Korotkoff sounds. b. Detect the presence of an auscultatory gap. c. Avoid missing a falsely elevated blood pressure. d. More readily identify phase IV of the Korotkoff sounds.

b. Detect the presence of an auscultatory gap. Rationale: Inflation of the cuff 20 to 30 mm Hg beyond the point at which a palpated pulse disappears will avoid missing an auscultatory gap, which is a period when the Korotkoff sounds disappear during auscultation.

A 69-year-old patient has been admitted to an adult psychiatric unit because his wife thinks he is getting more and more confused. He laughs when he is found to be forgetful, saying I'm just getting old! After the nurse completes a thorough neurologic assessment, which findings would be indicative of Alzheimer disease? Select all that apply. a. Occasionally forgetting names or appointments b. Difficulty performing familiar tasks, such as placing a telephone call c. Misplacing items, such as putting dish soap in the refrigerator d. Sometimes having trouble finding the right word e. Rapid mood swings, from calm to tears, for no apparent reason f. Getting lost in ones own neighborhood

b. Difficulty performing familiar tasks, such as placing a telephone call c. Misplacing items, such as putting dish soap in the refrigerator e. Rapid mood swings, from calm to tears, for no apparent reason f. Getting lost in ones own neighborhood Rationale: Difficulty performing familiar tasks, misplacing items, rapid mood swings, and getting lost in ones own neighborhood can be warning signs of Alzheimer disease. Occasionally forgetting names or appointments, and sometimes having trouble finding the right word are part of normal aging. (For other examples of Alzheimer disease, see Table 23-2.)

In obtaining a health history on a 74-year-old patient, the nurse notes that he drinks alcohol daily and that he has noticed a tremor in his hands that affects his ability to hold things. With this information, what response should the nurse make? a. Does your family know you are drinking every day? b. Does the tremor change when you drink alcohol? c. Well do some tests to see what is causing the tremor. d. You really shouldn't drink so much alcohol; it may be causing your tremor.

b. Does the tremor change when you drink alcohol? Rationale: Senile tremor is relieved by alcohol, although not a recommended treatment. The nurse should assess whether the person is abusing alcohol in an effort to relieve the tremor.

The nurse is assessing a patients skin during an office visit. What part of the hand and technique should be used to best assess the patients skin temperature? a. Fingertips; they are more sensitive to small changes in temperature. b. Dorsal surface of the hand; the skin is thinner on this surface than on the palms. c. Ulnar portion of the hand; increased blood supply in this area enhances temperature sensitivity. d. Palmar surface of the hand; this surface is the most sensitive to temperature variations because of its increased nerve supply in this area.

b. Dorsal surface of the hand; the skin is thinner on this surface than on the palms. Rationale: The dorsa (backs) of the hands and fingers are best for determining temperature because the skin is thinner on the dorsal surfaces than on the palms. Fingertips are best for fine, tactile discrimination. The other responses are not useful for palpation.

The nurse keeps in mind that the most important reason to share information and to offer brief teaching while performing the physical examination is to help the: a. Examiner feel more comfortable and to gain control of the situation. b. Examiner to build rapport and to increase the patients confidence in him or her. c. Patient understand his or her disease process and treatment modalities. d. Patient identify questions about his or her disease and the potential areas of patient education.

b. Examiner to build rapport and to increase the patients confidence in him or her. Rationale: Sharing information builds rapport and increases the patients confidence in the examiner. It also gives the patient a little more control in a situation during which feeling completely helpless is often present.

Which of the following statements is true regarding the internal structures of the breast? The breast is made up of: a. Primarily muscle with very little fibrous tissue. b. Fibrous, glandular, and adipose tissues. c. Primarily milk ducts, known as lactiferous ducts. d. Glandular tissue, which supports the breast by attaching to the chest wall.

b. Fibrous, glandular, and adipose tissues. Rationale: The breast is made up of glandular, fibrous (including the suspensory ligaments), and adipose tissues.

The nurse is preparing to conduct a mental status examination. Which statement is true regarding the mental status examination? a. A patients family is the best resource for information about the patients coping skills. b. Gathering mental status information during the health history interview is usually sufficient. c. Integrating the mental status examination into the health history interview takes an enormous amount of extra time. d. To get a good idea of the patients level of functioning, performing a complete mental status examination is usually necessary.

b. Gathering mental status information during the health history interview is usually sufficient. Rationale: The full mental status examination is a systematic check of emotional and cognitive functioning. The steps described, however, rarely need to be taken in their entirety. Usually, one can assess mental status through the context of the health history interview.

The nurse is assessing a 1-month-old infant at his well-baby checkup. Which assessment findings are appropriate for this age? Select all that apply. a. Head circumference equal to chest circumference b. Head circumference greater than chest circumference c. Head circumference less than chest circumference d. Fontanels firm and slightly concave e. Absent tonic neck reflex f. Nonpalpable cervical lymph nodes

b. Head circumference greater than chest circumference d. Fontanels firm and slightly concave f. Nonpalpable cervical lymph nodes Rationale: An infants head circumference is larger than the chest circumference. At age 2 years, both measurements are the same. During childhood, the chest circumference grows to exceed the head circumference by 5 to 7 cm. The fontanels should feel firm and slightly concave in the infant, and they should close by age 9 months. The tonic neck reflex is present until between 3 and 4 months of age, and cervical lymph nodes are normally nonpalpable in an infant.

An older patient has been admitted to the intensive care unit (ICU) after falling at home. Within 8 hours, his condition has stabilized and he is transferred to a medical unit. The family is wondering whether he will be able to go back home. Which assessment instrument is most appropriate for the nurse to choose at this time? a. Lawton IADL instrument b. Hospital Admission Risk Profile (HARP) c. Mini-Cog d. NEECHAM Confusion Scale

b. Hospital Admission Risk Profile (HARP) Rationale: Hospital-acquired functional decline may occur within 2 days of a hospital admission. The HARP helps identify older adults who are at greatest risk of losing their ability to perform ADLs or mobility at this critical time. The Lawton IADL measures instrumental activities of daily living, which may be difficult to observe in the hospital setting. The Mini-Cog is an assessment of mental status. The NEECHAM Confusion Scale is used to assess for delirium.

When beginning to assess a persons spirituality, which question by the nurse would be most appropriate? a. Do you believe in God? b. How does your spirituality relate to your health care decisions? c. What religious faith do you follow? d. Do you believe in the power of prayer?

b. How does your spirituality relate to your health care decisions? Rationale: Open-ended questions provide a foundation for future discussions. The other responses are easily answered by one-word replies and are closed questions.

The nurse is conducting a developmental history on a 5-year-old child. Which questions are appropriate to ask the parents for this part of the assessment? Select all that apply. a. How much junk food does your child eat? b. How many teeth has he lost, and when did he lose them? c. Is he able to tie his shoelaces? d. Does he take a childrens vitamin? e. Can he tell time? f. Does he have any food allergies?

b. How many teeth has he lost, and when did he lose them? c. Is he able to tie his shoelaces? e. Can he tell time? Rationale: Questions about tooth loss, ability to tell time, and ability to tie shoelaces are appropriate questions for a developmental assessment. Questions about junk food intake and vitamins are part of a nutritional history. Questions about food allergies are not part of a developmental history.

The nurse is examining a 62-year-old man and notes that he has bilateral gynecomastia. The nurse should explore his health history for which related conditions? Select all that apply. a. Malnutrition b. Hyperthyroidism c. Type 2 diabetes mellitus d. Liver disease e. History of alcohol abuse

b. Hyperthyroidism d. Liver disease e. History of alcohol abuse Rationale: Gynecomastia occurs with Cushing syndrome, liver cirrhosis, adrenal disease, hyperthyroidism, and numerous drugs, such as alcohol and marijuana use, estrogen treatment for prostate cancer, antibiotics (metronidazole, isoniazid), digoxin, angiotensin-converting enzyme (ACE) inhibitors, diazepam, and tricyclic antidepressants.

During a physical education class, a student is hit in the eye with the end of a baseball bat. When examined in the emergency department, the nurse notices the presence of blood in the anterior chamber of the eye. This finding indicates the presence of: a. Hypopyon. b. Hyphema. c. Corneal abrasion. d. Pterygium.

b. Hyphema. Rationale: Hyphema is the term for blood in the anterior chamber and is a serious result of blunt trauma (a fist or a baseball) or spontaneous hemorrhage and may indicate scleral rupture or major intraocular trauma. (See Table 14-7 for descriptions of the other terms.)

If a patient reports a recent breast infection, then the nurse should expect to find ________ node enlargement. a. Nonspecific b. Ipsilateral axillary c. Contralateral axillary d. Inguinal and cervical

b. Ipsilateral axillary Rationale: The breast has extensive lymphatic drainage. Most of the lymph, more than 75%, drains into the ipsilateral, or same side, axillary nodes.

A patient says that she has recently noticed a lump in the front of her neck below her Adams apple that seems to be getting bigger. During the assessment, the finding that leads the nurse to suspect that this may not be a cancerous thyroid nodule is that the lump (nodule): a. Is tender. b. Is mobile and not hard. c. Disappears when the patient smiles. d. Is hard and fixed to the surrounding structures.

b. Is mobile and not hard. Rationale: Painless, rapidly growing nodules may be cancerous, especially the appearance of a single nodule in a young person. However, cancerous nodules tend to be hard and fixed to surrounding structures, not mobile.

Which of these actions illustrates the correct technique the nurse should use when assessing oral temperature with a mercury thermometer? a. Wait 30 minutes if the patient has ingested hot or iced liquids. b. Leave the thermometer in place 3 to 4 minutes if the patient is afebrile. c. Place the thermometer in front of the tongue, and ask the patient to close his or her lips. d. Shake the mercury-in-glass thermometer down to below 36.6 C before taking the temperature.

b. Leave the thermometer in place 3 to 4 minutes if the patient is afebrile. Rationale: The thermometer should be left in place 3 to 4 minutes if the person is afebrile and up to 8 minutes if the person is febrile. The nurse should wait 15 minutes if the person has just ingested hot or iced liquids and 2 minutes if he or she has just smoked.

During the neurologic assessment of a healthy 35-year-old patient, the nurse asks him to relax his muscles completely. The nurse then moves each extremity through full range of motion. Which of these results would the nurse expect to find? a. Firm, rigid resistance to movement b. Mild, even resistance to movement c. Hypotonic muscles as a result of total relaxation d. Slight pain with some directions of movement

b. Mild, even resistance to movement Rationale: Tone is the normal degree of tension (contraction) in voluntarily relaxed muscles. It shows a mild resistance to passive stretching. Normally, the nurse will notice a mild, even resistance to movement. The other responses are not correct.

When auscultating the anterior thorax of a pregnant woman, the nurse notices the presence of a murmur over the second, third, and fourth intercostal spaces. The murmur is continuous but can be obliterated by pressure with the stethoscope or finger on the thorax just lateral to the murmur. The nurse interprets this finding to be: a. Murmur of aortic stenosis. b. Most likely a mammary souffle. c. Associated with aortic insufficiency. d. Indication of a patent ductus arteriosus.

b. Most likely a mammary souffle. Rationale: Blood flow through the blood vessels, specifically the internal mammary artery, can often be heard over the second, third, and fourth intercostal spaces. This finding is called a mammary souffle, but it may be mistaken for a cardiac murmur. The other options are incorrect.

During an assessment of the CNs, the nurse finds the following: asymmetry when the patient smiles or frowns, uneven lifting of the eyebrows, sagging of the lower eyelids, and escape of air when the nurse presses against the right puffed cheek. This would indicate dysfunction of which of these CNs? a. Motor component of CN IV b. Motor component of CN VII c. Motor and sensory components of CN XI d. Motor component of CN X and sensory component of CN VII

b. Motor component of CN VII Rationale: The findings listed reflect a dysfunction of the motor component of the facial nerve (CN VII).

A patient states during the interview that she noticed a new lump in the shower a few days ago. It was on her left breast near her axilla. The nurse should plan to: a. Palpate the lump first. b. Palpate the unaffected breast first. c. Avoid palpating the lump because it could be a cyst, which might rupture. d. Palpate the breast with the lump first but plan to palpate the axilla last.

b. Palpate the unaffected breast first. Rationale: If the woman mentions a breast lump she has discovered herself, then the nurse should examine the unaffected breast first to learn a baseline of normal consistency for this individual.

During a breast examination on a female patient, the nurse notices that the nipple is flat, broad, and fixed. The patient states it started doing that a few months ago. This finding suggests: a. Dimpling. b. Retracted nipple. c. Nipple inversion. d. Deviation in nipple pointing.

b. Retracted nipple. Rationale: The retracted nipple looks flatter and broader, similar to an underlying crater. A recent retraction suggests cancer, which causes fibrosis of the whole duct system and pulls in the nipple. It also may occur with benign lesions such as ectasia of the ducts. The nurse should not confuse retraction with the normal long-standing type of nipple inversion, which has no broadening and is not fixed.

A patient has been admitted to a hospital after the staff in the nursing home noticed a pressure ulcer in his sacral area. The nurse examines the pressure ulcer and determines that it is a stage II ulcer. Which of these findings are characteristic of a stage II pressure ulcer? Select all that apply. a. Intact skin appears red but is not broken. b. Partial thickness skin erosion is observed with a loss of epidermis or dermis. c. Ulcer extends into the subcutaneous tissue. d. Localized redness in light skin will blanch with fingertip pressure. e. Open blister areas have a red-pink wound bed. f. Patches of eschar cover parts of the wound.

b. Partial thickness skin erosion is observed with a loss of epidermis or dermis. e. Open blister areas have a red-pink wound bed. Rationale: Stage I pressure ulcers have intact skin that appears red but is not broken, and localized redness in intact skin will blanche with fingertip pressure. Stage II pressure ulcers have partial thickness skin erosion with a loss of epidermis or also the dermis; open blisters have a red-pink wound bed. Stage III pressure ulcers are full thickness, extending into the subcutaneous tissue; subcutaneous fat may be seen but not muscle, bone, or tendon. Stage IV pressure ulcers involve all skin layers and extend into supporting tissue, exposing muscle, bone, and tendon. Slough (stringy matter attached to the wound bed) or eschar (black or brown necrotic tissue) may be present.

During an examination, a patient states that she was diagnosed with open-angle glaucoma 2 years ago. The nurse assesses for characteristics of open-angle glaucoma. Which of these are characteristics of open-angle glaucoma? Select all that apply. a. Patient may experience sensitivity to light, nausea, and halos around lights. b. Patient experiences tunnel vision in the late stages. c. Immediate treatment is needed. d. Vision loss begins with peripheral vision. e. Open-angle glaucoma causes sudden attacks of increased pressure that cause blurred vision. f. Virtually no symptoms are exhibited.

b. Patient experiences tunnel vision in the late stages. d. Vision loss begins with peripheral vision. f. Virtually no symptoms are exhibited. Rationale: Open-angle glaucoma is the most common type of glaucoma; virtually no symptoms are exhibited. Vision loss begins with the peripheral vision, which often goes unnoticed because individuals learn to compensate intuitively by turning their heads. The other characteristics are those of closed-angle glaucoma.

The nurse is assessing the neurologic status of a patient who has a late-stage brain tumor. With the reflex hammer, the nurse draws a light stroke up the lateral side of the sole of the foot and inward, across the ball of the foot. In response, the patients toes fan out, and the big toe shows dorsiflexion. The nurse interprets this result as: a. Negative Babinski sign, which is normal for adults. b. Positive Babinski sign, which is abnormal for adults. c. Clonus, which is a hyperactive response. d. Achilles reflex, which is an expected response.

b. Positive Babinski sign, which is abnormal for adults. Rationale: Dorsiflexion of the big toe and fanning of all toes is a positive Babinski sign, also called up-going toes. This response occurs with upper motor neuron disease of the corticospinal (or pyramidal) tract and is an abnormal finding for adults.

The nurse is assessing the abilities of an older adult. Which activities are considered IADLs? Select all that apply. a. Feeding oneself b. Preparing a meal c. Balancing a checkbook d. Walking e. Toileting f. Grocery shopping

b. Preparing a meal c. Balancing a checkbook f. Grocery shopping Rationale: Typically, IADL tasks include shopping, meal preparation, housekeeping, laundry, managing finances, taking medications, and using transportation. The other options listed are ADLs related to self-care.

The nurse is testing the hearing of a 78-year-old man and is reminded of the changes in hearing that occur with aging that include which of the following? Select all that apply. a. Hearing loss related to aging begins in the mid 40s. b. Progression of hearing loss is slow. c. The aging person has low-frequency tone loss. d. The aging person may find it harder to hear consonants than vowels. e. Sounds may be garbled and difficult to localize. f. Hearing loss reflects nerve degeneration of the middle ear.

b. Progression of hearing loss is slow. d. The aging person may find it harder to hear consonants than vowels. e. Sounds may be garbled and difficult to localize. Rationale: Presbycusis is a type of hearing loss that occurs with aging and is found in 60% of those older than 65 years. It is a gradual sensorineural loss caused by nerve degeneration in the inner ear or auditory nerve, and it slowly progresses after the age of 50 years. The person first notices a high-frequency tone loss; it is harder to hear consonants (high-pitched components of speech) than vowels, which makes words sound garbled. The ability to localize sound is also impaired.

The physician reports that a patient with a neck tumor has a tracheal shift. The nurse is aware that this means that the patients trachea is: a. Pulled to the affected side. b. Pushed to the unaffected side. c. Pulled downward. d. Pulled downward in a rhythmic pattern.

b. Pushed to the unaffected side. Rationale: The trachea is pushed to the unaffected side with an aortic aneurysm, a tumor, unilateral thyroid lobe enlargement, or a pneumothorax. The trachea is pulled to the affected side with large atelectasis, pleural adhesions, or fibrosis. Tracheal tug is a rhythmic downward pull that is synchronous with systole and occurs with aortic arch aneurysm.

The nurse is reviewing aspects of substance abuse in preparation for a seminar. Which of these statements illustrates the concept of tolerance to an illicit substance? The person: a. Has a physiologic dependence on a substance. b. Requires an increased amount of the substance to produce the same effect. c. Requires daily use of the substance to function and is unable to stop using it. d. Experiences a syndrome of physiologic symptoms if the substance is not used.

b. Requires an increased amount of the substance to produce the same effect. Rationale: The concept of tolerance to a substance indicates that the person requires an increased amount of the substance to produce the same effect. Abuse indicates that the person needs to use the substance daily to function, and the person is unable to stop using it. Dependence is an actual physiologic dependence on the substance. Withdrawal occurs when cessation of the substance produces a syndrome of physiologic symptoms.

A patient comes to the emergency department after a boxing match, and his left eye is swollen almost shut. He has bruises on his face and neck. He says he is worried because he cant see well from his left eye. The physician suspects retinal damage. The nurse recognizes that signs of retinal detachment include: a. Loss of central vision. b. Shadow or diminished vision in one quadrant or one half of the visual field. c. Loss of peripheral vision. d. Sudden loss of pupillary constriction and accommodation.

b. Shadow or diminished vision in one quadrant or one half of the visual field. Rationale: With retinal detachment, the person has shadows or diminished vision in one quadrant or one half of the visual field. The other responses are not signs of retinal detachment.

The nurse has completed an assessment on a patient who came to the clinic for a leg injury. As a result of the assessment, the nurse has determined that the patient has at-risk alcohol use. Which action by the nurse is most appropriate at this time? a. Record the results of the assessment, and notify the physician on call. b. State, You are drinking more than is medically safe. I strongly recommend that you quit drinking, and I'm willing to help you. c. State, It appears that you may have a drinking problem. Here is the telephone number of our local Alcoholics Anonymous chapter. d. Give the patient information about a local rehabilitation clinic.

b. State, You are drinking more than is medically safe. I strongly recommend that you quit drinking, and I'm willing to help you. Rationale: If an assessment has determined that the patient has at-risk drinking behavior, then the nurse should give a short but clear statement of assistance and concern. Simply giving out a telephone number or referral to agencies may not be enough.

A patient is being seen at the clinic for her 10-week prenatal visit. She asks when she will be able to hear the baby's heartbeat. The nurse should reply: a. The baby's heartbeat is not usually heard until the second trimester. b. The baby's heartbeat may be heard anywhere from the ninth to the twelfth week. c. It is often difficult to hear the heartbeat at this point, but we can try. d. It is normal to hear the heartbeat at 6 weeks. We may be able to hear it today.

b. The baby's heartbeat may be heard anywhere from the ninth to the twelfth week. Rationale: Fetal heart tones can be heard with the use of the Doppler device between 9 and 12 weeks. The other responses are incorrect.

Which statement concerning the areas of the brain is true? a. The cerebellum is the center for speech and emotions. b. The hypothalamus controls body temperature and regulates sleep. c. The basal ganglia are responsible for controlling voluntary movements. d. Motor pathways of the spinal cord and brainstem synapse in the thalamus.

b. The hypothalamus controls body temperature and regulates sleep. Rationale: The hypothalamus is a vital area with many important functions: body temperature controller, sleep center, anterior and posterior pituitary gland regulator, and coordinator of autonomic nervous system activity and emotional status. The cerebellum controls motor coordination, equilibrium, and balance. The basal ganglia control autonomic movements of the body. The motor pathways of the spinal cord synapse in various areas of the spinal cord, not in the thalamus.

The nurse is conducting a visual examination. Which of these statements regarding visual pathways and visual fields is true? a. The right side of the brain interprets the vision for the right eye. b. The image formed on the retina is upside down and reversed from its actual appearance in the outside world. c. Light rays are refracted through the transparent media of the eye before striking the pupil. d. Light impulses are conducted through the optic nerve to the temporal lobes of the brain.

b. The image formed on the retina is upside down and reversed from its actual appearance in the outside world. Rationale: The image formed on the retina is upside down and reversed from its actual appearance in the outside world. The light rays are refracted through the transparent media of the eye before striking the retina, and the nerve impulses are conducted through the optic nerve tract to the visual cortex of the occipital lobe of the brain. The left side of the brain interprets vision for the right eye.

In performing a breast examination, the nurse knows that examining the upper outer quadrant of the breast is especially important. The reason for this is that the upper outer quadrant is: a. The largest quadrant of the breast. b. The location of most breast tumors. c. Where most of the suspensory ligaments attach. d. More prone to injury and calcifications than other locations in the breast.

b. The location of most breast tumors. Rationale: The upper outer quadrant is the site of most breast tumors. In the upper outer quadrant, the nurse should notice the axillary tail of Spence, the cone-shaped breast tissue that projects up into the axilla, close to the pectoral group of axillary lymph nodes.

A patients vision is recorded as 20/30 when the Snellen eye chart is used. The nurse interprets these results to indicate that: a. At 30 feet the patient can read the entire chart. b. The patient can read at 20 feet what a person with normal vision can read at 30 feet. c. The patient can read the chart from 20 feet in the left eye and 30 feet in the right eye. d. The patient can read from 30 feet what a person with normal vision can read from 20 feet.

b. The patient can read at 20 feet what a person with normal vision can read at 30 feet. Rationale: The top number indicates the distance the person is standing from the chart; the denominator gives the distance at which a normal eye can see.

The nurse is conducting an interview. Which of these statements is true regarding open-ended questions? Select all that apply. a. Open-ended questions elicit cold facts b. They allow for self-expression c. Open-ended questions build and enhance rapport d. They leave interactions neutral e. Open-ended questions call for short one- to two-word answers f. They are used when narrative information is needed

b. They allow for self-expression c. Open-ended questions build and enhance rapport f. They are used when narrative information is needed Rationale: Open-ended questions allow for self-expression, build and enhance rapport, and obtain narrative information. These features enhance communication during an interview. The other statements are appropriate for closed or direct questions.

A 17-year-old student is a swimmer on her high schools swim team. She has had three bouts of otitis externa this season and wants to know what to do to prevent it. The nurse instructs her to: a. Use a cotton-tipped swab to dry the ear canals thoroughly after each swim. b. Use rubbing alcohol or 2% acetic acid eardrops after every swim. c. Irrigate the ears with warm water and a bulb syringe after each swim. d. Rinse the ears with a warmed solution of mineral oil and hydrogen peroxide.

b. Use rubbing alcohol or 2% acetic acid eardrops after every swim. Rationale: With otitis externa (swimmers ear), swimming causes the external canal to become waterlogged and swell; skinfolds are set up for infection. Otitis externa can be prevented by using rubbing alcohol or 2% acetic acid eardrops after every swim.

A patient comes to the clinic complaining of neck and shoulder pain and is unable to turn her head. The nurse suspects damage to CN ______ and proceeds with the examination by _____________. a. XI; palpating the anterior and posterior triangles b. XI; asking the patient to shrug her shoulders against resistance c. XII; percussing the sternomastoid and submandibular neck muscles d. XII; assessing for a positive Romberg sign

b. XI; asking the patient to shrug her shoulders against resistance Rationale: The major neck muscles are the sternomastoid and the trapezius. They are innervated by CN XI, the spinal accessory. The innervated muscles assist with head rotation and head flexion, movement of the shoulders, and extension and turning of the head.

A 2-year-old boy has been diagnosed with physiologic cryptorchidism. Considering this diagnosis, during assessment the nurse will most likely observe: a. Testes that are hard and painful to palpation. b. Atrophic scrotum and a bilateral absence of the testis. c. Absence of the testis in the scrotum, but the testis can be milked down. d. Testes that migrate into the abdomen when the child squats or sits cross-legged.

c. Absence of the testis in the scrotum, but the testis can be milked down. Rationale: Migratory testes (physiologic cryptorchidism) are common because of the strength of the cremasteric reflex and the small mass of the prepubertal testes. The affected side has a normally developed scrotum and the testis can be milked down. The other responses are not correct.

A patient is being assessed for range-of-joint movement. The nurse asks him to move his arm in toward the center of his body. This movement is called: a. Flexion. b. Abduction. c. Adduction. d. Extension.

c. Adduction. Rationale: Moving a limb toward the midline of the body is called adduction; moving a limb away from the midline of the body is called abduction. Flexion is bending a limb at a joint; and extension is straightening a limb at a joint.

The nurse is palpating the fundus of a pregnant woman. Which statement about palpation of the fundus is true? a. The fundus should be hard and slightly tender to palpation during the first trimester. b. Fetal movement may not be felt by the examiner until the end of the second trimester. c. After 20 weeks gestation, the number of centimeters should approximate the number of weeks gestation. d. Fundal height is usually less than the number of weeks gestation, unless an abnormal condition such as excessive amniotic fluid is present.

c. After 20 weeks gestation, the number of centimeters should approximate the number of weeks gestation. Rationale: After 20 weeks gestation, the number of centimeters should approximate the number of weeks gestation. In addition, at 20 weeks gestation, the examiner may be able to feel fetal movement and the head can be balloted.

The nurse is preparing to assess an older adult and discovers that the older adult is in severe pain. Which statement about pain and the older adult is true? a. Pain is inevitable with aging. b. Older adults with cognitive impairments feel less pain. c. Alleviating pain should be a priority over other aspects of the assessment. d. The assessment should take priority so that care decisions can be made.

c. Alleviating pain should be a priority over other aspects of the assessment. Rationale: If the older adult is experiencing pain or discomfort, then the depth of knowledge gathered through the assessments will suffer. Alleviating pain should be a priority over other aspects of the assessment. Remembering that older adults with cognitive impairment do not feel less pain is paramount.

A 30-year-old female patient is describing feelings of hopelessness and depression. She has attempted self-mutilation and has a history of suicide attempts. She describes difficulty sleeping at night and has lost 10 pounds in the past month. Which of these statements or questions is the nurses best response in this situation? a. Do you have a weapon? b. How do other people treat you? c. Are you feeling so hopeless that you feel like hurting yourself now? d. People often feel hopeless, but the feelings resolve within a few weeks.

c. Are you feeling so hopeless that you feel like hurting yourself now? Rationale: When the person expresses feelings of hopelessness, despair, or grief, assessing the risk of physical harm to him or herself is important. This process begins with more general questions. If the answers are affirmative, then the assessment continues with more specific questions.

During a health history interview, a female patient states that she has noticed a few drops of clear discharge from her right nipple. What should the nurse do next? a. Immediately contact the physician to report the discharge. b. Ask her if she is possibly pregnant. c. Ask the patient some additional questions about the medications she is taking. d. Immediately obtain a sample for culture and sensitivity testing.

c. Ask the patient some additional questions about the medications she is taking. Rationale: The use of some medications, such as oral contraceptives, phenothiazines, diuretics, digitalis, steroids, methyldopa, and calcium channel blockers, may cause clear nipple discharge. Bloody or blood-tinged discharge from the nipple, not clear, is significant, especially if a lump is also present. In the pregnant female, colostrum would be a thick, yellowish liquid, and it would be normally expressed after the fourth month of pregnancy.

A physician tells the nurse that a patients vertebra prominens is tender and asks the nurse to reevaluate the area in 1 hour. The area of the body the nurse will assess is: a. Just above the diaphragm. b. Just lateral to the knee cap. c. At the level of the C7 vertebra. d. At the level of the T11 vertebra.

c. At the level of the C7 vertebra. Rationale: The C7 vertebra has a long spinous process, called the vertebra prominens, which is palpable when the head is flexed.

The nurse needs to assess a patients ability to perform activities of daily living (ADLs) and should choose which tool for this assessment? a. Direct Assessment of Functional Abilities (DAFA) b. Lawton Instrumental Activities of Daily Living (IADL) scale c. Barthel Index d. Older Americans Resources and Services Multidimensional Functional Assessment QuestionnaireIADL (OMFAQ-IADL)

c. Barthel Index Rationale: The Barthel Index is used to assess ADLs. The other options are used to measure IADLs.

A woman is in the family planning clinic seeking birth control information. She states that her breasts change all month long and that she is worried that this is unusual. What is the nurses best response? The nurse should tell her that: a. Continual changes in her breasts are unusual. The breasts of nonpregnant women usually stay pretty much the same all month long. b. Breast changes in response to stress are very common and that she should assess her life for stressful events. c. Because of the changing hormones during the monthly menstrual cycle, cyclic breast changes are common. d. Breast changes normally occur only during pregnancy and that a pregnancy test is needed at this time.

c. Because of the changing hormones during the monthly menstrual cycle, cyclic breast changes are common. Rationale: Breasts of the nonpregnant woman change with the ebb and flow of hormones during the monthly menstrual cycle. During the 3 to 4 days before menstruation, the breasts feel full, tight, heavy, and occasionally sore. The breast volume is smallest on days 4 to 7 of the menstrual cycle.

The nurse is preparing for a class in early detection of breast cancer. Which statement is true with regard to breast cancer in black women in the United States? a. Breast cancer is not a threat to black women. b. Black women have a lower incidence of regional or distant breast cancer than white women. c. Black women are more likely to die of breast cancer at any age. d. Breast cancer incidence in black women is higher than that of white women after age 45.

c. Black women are more likely to die of breast cancer at any age. Rationale: Black women have a higher incidence of breast cancer before age 45 years than white women and are more likely to die of their disease. In addition, black women are significantly more likely to be diagnosed with regional or distant breast cancer than are white women. These racial differences in mortality rates may be related to an insufficient use of screening measures and a lack of access to health care.

In performing an assessment of a womans axillary lymph system, the nurse should assess which of these nodes? a. Central, axillary, lateral, and sternal b. Pectoral, lateral, anterior, and sternal c. Central, lateral, pectoral, and subscapular d. Lateral, pectoral, axillary, and suprascapular

c. Central, lateral, pectoral, and subscapular Rationale: The breast has extensive lymphatic drainage. Four groups of axillary nodes are present: (1) central, (2) pectoral (anterior), (3) subscapular (posterior), and (4) lateral.

A 30-year-old woman tells the nurse that she has been very unsteady and has had difficulty in maintaining her balance. Which area of the brain that is related to these findings would concern the nurse? a. Thalamus b. Brainstem c. Cerebellum d. Extrapyramidal tract

c. Cerebellum Rationale: The cerebellar system coordinates movement, maintains equilibrium, and helps maintain posture. The thalamus is the primary relay station where sensory pathways of the spinal cord, cerebellum, and brainstem form synapses on their way to the cerebral cortex. The brainstem consists of the midbrain, pons, and medulla and has various functions, especially concerning autonomic centers. The extrapyramidal tract maintains muscle tone for gross automatic movements, such as walking.

A patient repeatedly seems to have difficulty coming up with a word. He says, I was on my way to work, and when I got there, the thing that you step into that goes up in the air was so full that I decided to take the stairs. The nurse will note on his chart that he is using or experiencing: a. Blocking b. Neologism c. Circumlocution d. Circumstantiality

c. Circumlocution Rationale: Circumlocution is a roundabout expression, substituting a phrase when one cannot think of the name of the object.

During an admission assessment, the nurse notices that a male patient has an enlarged and rather thick skull. The nurse suspects acromegaly and would further assess for: a. Exophthalmos. b. Bowed long bones. c. Coarse facial features. d. Acorn-shaped cranium.

c. Coarse facial features. Rationale: Acromegaly is excessive secretion of growth hormone that creates an enlarged skull and thickened cranial bones. Patients will have elongated heads, massive faces, prominent noses and lower jaws, heavy eyebrow ridges, and coarse facial features. Exophthalmos is associated with hyperthyroidism. Bowed long bones and an acorn-shaped cranium result from Paget disease.

The nurse is preparing to percuss the abdomen of a patient. The purpose of the percussion is to assess the __________ of the underlying tissue. a. Turgor b. Texture c. Density d. Consistency

c. Density Rationale: Percussion yields a sound that depicts the location, size, and density of the underlying organ. Turgor and texture are assessed with palpation.

The nurse is performing a genitourinary assessment on a 50-year-old obese male laborer. On examination, the nurse notices a painless round swelling close to the pubis in the area of the internal inguinal ring that is easily reduced when the individual is supine. These findings are most consistent with a(n) ______ hernia. a. Scrotal b. Femoral c. Direct inguinal d. Indirect inguinal

c. Direct inguinal Rationale: Direct inguinal hernias occur most often in men over the age of 40 years. It is an acquired weakness brought on by heavy lifting, obesity, chronic cough, or ascites. The direct inguinal hernia is usually a painless, round swelling close to the pubis in the area of the internal inguinal ring that is easily reduced when the individual is supine. (See Table 24-6 for a description of scrotal hernia. See Table 24-7 for the descriptions of femoral hernias and indirect inguinal hernias.)

When measuring a patients body temperature, the nurse keeps in mind that body temperature is influenced by: a. Constipation. b. Patients emotional state. c. Diurnal cycle. d. Nocturnal cycle.

c. Diurnal cycle. Rationale: Normal temperature is influenced by the diurnal cycle, exercise, and age. The other responses do not influence body temperature.

During a discussion about BSEs with a 30-year-old woman, which of these statements by the nurse is most appropriate? a. The best time to examine your breasts is during ovulation. b. Examine your breasts every month on the same day of the month. c. Examine your breasts shortly after your menstrual period each month. d. The best time to examine your breasts is immediately before menstruation.

c. Examine your breasts shortly after your menstrual period each month. Rationale: The best time to conduct a BSE is shortly after the menstrual period when the breasts are the smallest and least congested.

In examining a 70-year-old male patient, the nurse notices that he has bilateral gynecomastia. Which of the following describes the nurses best course of action? a. Recommend that he make an appointment with his physician for a mammogram. b. Ignore it. Benign breast enlargement in men is not unusual. c. Explain that this condition may be the result of hormonal changes, and recommend that he see his physician. d. Explain that gynecomastia in men is usually associated with prostate enlargement and recommend that he be thoroughly screened.

c. Explain that this condition may be the result of hormonal changes, and recommend that he see his physician. Rationale: Gynecomastia may reappear in the aging man and may be attributable to a testosterone deficiency.

When the nurse asks for a description of who lives with a child, the method of discipline, and the support system of the child, what part of the assessment is being performed? a. Family history b. Review of systems c. Functional assessment d. Reason for seeking care

c. Functional assessment Rationale: Functional assessment includes interpersonal relationships and home environment. Family history includes illnesses in family members; a review of systems includes questions about the various body systems; and the reason for seeking care is the rationale for requesting health care.

During a genital examination, the nurse notices that a male patient has clusters of small vesicles on the glans, surrounded by erythema. The nurse recognizes that these lesions are: a. Peyronie disease. b. Genital warts. c. Genital herpes. d. Syphilitic cancer.

c. Genital herpes. Rationale: Genital herpes, or herpes simplex virus 2 (HSV-2), infections are indicated with clusters of small vesicles with surrounding erythema, which are often painful and erupt on the glans or foreskin. (See Table 24-4 for the descriptions of the other options.)

During an examination of a patient in her third trimester of pregnancy, the nurse notices that the patients thyroid gland is slightly enlarged. No enlargement had been previously noticed. The nurse suspects that the patient: a. Has an iodine deficiency. b. Is exhibiting early signs of goiter. c. Is exhibiting a normal enlargement of the thyroid gland during pregnancy. d. Needs further testing for possible thyroid cancer.

c. Is exhibiting a normal enlargement of the thyroid gland during pregnancy. Rationale: The thyroid gland enlarges slightly during pregnancy because of hyperplasia of the tissue and increased vascularity.

When assessing the pupillary light reflex, the nurse should use which technique? a. Shine a penlight from directly in front of the patient, and inspect for pupillary constriction. b. Ask the patient to follow the penlight in eight directions, and observe for bilateral pupil constriction. c. Shine a light across the pupil from the side, and observe for direct and consensual pupillary constriction. d. Ask the patient to focus on a distant object. Then ask the patient to follow the penlight to approximately 7 cm from the nose.

c. Shine a light across the pupil from the side, and observe for direct and consensual pupillary constriction. Rationale: To test the pupillary light reflex, the nurse should advance a light in from the side and note the direct and consensual pupillary constriction.

During a group prenatal teaching session, the nurse teaches Kegel exercises. Which statements would be appropriate for this teaching session? Select all that apply. a. Kegel exercises help keep your uterus strong during the pregnancy. b. Kegel exercises should be performed twice a day. c. Kegel exercises should be performed 50 to 100 times a day. d. To perform Kegel exercises, slowly squeeze to a peak at the count of eight, and then slowly release to a count of eight. e. To perform Kegel exercises, rapidly perform alternating squeeze-release exercises up to the count of eight.

c. Kegel exercises should be performed 50 to 100 times a day. d. To perform Kegel exercises, slowly squeeze to a peak at the count of eight, and then slowly release to a count of eight. Rationale: Kegel exercises can be performed to prepare for and to recover from birth. The nurse should direct the woman to squeeze slowly to a peak at the count of eight and then to release slowly to the count of eight. The nurse can prescribe this exercise to be performed 50 to 100 times a day.

The nurse auscultates a functional systolic murmur, grade II/IV, on a woman in week 30 of her pregnancy. The remainder of her physical assessment is within normal limits. The nurse would: a. Consider this finding abnormal, and refer her for additional consultation. b. Ask the woman to run briefly in place and then assess for an increase in intensity of the murmur. c. Know that this finding is normal and is a result of the increase in blood volume during pregnancy. d. Ask the woman to restrict her activities and return to the clinic in 1 week for re-evaluation.

c. Know that this finding is normal and is a result of the increase in blood volume during pregnancy. Rationale: Because of the increase in blood volume, a functional systolic murmur, grade II/IV or less, can be heard in 95% of pregnant women. The other actions are not appropriate.

While gathering equipment after an injection, a nurse accidentally received a prick from an improperly capped needle. To interpret this sensation, which of these areas must be intact? a. Corticospinal tract, medulla, and basal ganglia b. Pyramidal tract, hypothalamus, and sensory cortex c. Lateral spinothalamic tract, thalamus, and sensory cortex d. Anterior spinothalamic tract, basal ganglia, and sensory cortex

c. Lateral spinothalamic tract, thalamus, and sensory cortex Rationale: The spinothalamic tract contains sensory fibers that transmit the sensations of pain, temperature, and crude or light touch. Fibers carrying pain and temperature sensations ascend the lateral spinothalamic tract, whereas the sensations of crude touch form the anterior spinothalamic tract. At the thalamus, the fibers synapse with another sensory neurons, which carries the message to the sensory cortex for full interpretation. The other options are not correct.

A patients pregnancy test is positive, and she wants to know when the baby is due. The first day of her last menstrual period was June 14, and that period ended June 20. Using the Nagele rule, what is her expected date of delivery? a. March 7 b. March 14 c. March 21 d. March 27

c. March 21 Rationale: To determine the expected date of delivery using the Nagele rule, 7 days are added to the first day of the last menstrual period; then 3 months are subtracted. Therefore, adding 7 days to June 14 would be June 21 and subtracting 3 months would make the expected delivery date March 21.

The nurse is preparing to measure the length, weight, chest, and head circumference of a 6-month-old infant. Which measurement technique is correct? a. Measuring the infants length by using a tape measure b. Weighing the infant by placing him or her on an electronic standing scale c. Measuring the chest circumference at the nipple line with a tape measure d. Measuring the head circumference by wrapping the tape measure over the nose and cheekbones

c. Measuring the chest circumference at the nipple line with a tape measure Rationale: To measure the chest circumference, the tape is encircled around the chest at the nipple line. The length should be measured on a horizontal measuring board. Weight should be measured on a platform-type balance scale. Head circumference is measured with the tape around the head, aligned at the eyebrows, and at the prominent frontal and occipital bonesthe widest span is correct.

The nurse is aware of which statement to be true regarding the incidence of testicular cancer? a. Testicular cancer is the most common cancer in men aged 30 to 50 years. b. The early symptoms of testicular cancer are pain and induration. c. Men with a history of cryptorchidism are at the greatest risk for the development of testicular cancer. d. The cure rate for testicular cancer is low.

c. Men with a history of cryptorchidism are at the greatest risk for the development of testicular cancer. Rationale: Men with undescended testicles (cryptorchidism) are at the greatest risk for the development of testicular cancer. The overall incidence of testicular cancer is rare. Although testicular cancer has no early symptoms, when detected early and treated before metastasizing, the cure rate is almost 100%.

The nurse is performing a mental status examination. Which statement is true regarding the assessment of mental status? a. Mental status assessment diagnoses specific psychiatric disorders. b. Mental disorders occur in response to everyday life stressors. c. Mental status functioning is inferred through the assessment of an individuals behaviors. d. Mental status can be directly assessed, similar to other systems of the body (e.g., heart sounds, breath sounds).

c. Mental status functioning is inferred through the assessment of an individuals behaviors. Rationale: Mental status functioning is inferred through the assessment of an individuals behaviors. It cannot be directly assessed like the characteristics of the skin or heart sounds.

During a morning assessment, the nurse notices that an older patient is less attentive and is unable to recall yesterdays events. Which test is appropriate for assessing the patients mental status? a. Geriatric Depression Scale, short form b. Rapid Disability Rating Scale-2 c. Mini-Cog d. Get Up and Go Test

c. Mini-Cog Rationale: For nurses in various settings, cognitive assessments provide continuing comparisons to the individuals baseline to detect any acute changes in mental status. The Mini-Cog is a mental status test that tests immediate and delayed recall and visuospatial abilities. The Geriatric Depression Scale, short form, assesses for depression and changes in the level of depression, not mental status. The Rapid Disability Rating Scale-2 measures what the person can actually do versus what he or she could do, but not mental status. The Get Up and Go Test assesses functional mobility, not mental status.

A patient, an 85-year-old woman, is complaining about the fact that the bones in her face have become more noticeable. What explanation should the nurse give her? a. Diets low in protein and high in carbohydrates may cause enhanced facial bones. b. Bones can become more noticeable if the person does not use a dermatologically approved moisturizer. c. More noticeable facial bones are probably due to a combination of factors related to aging, such as decreased elasticity, subcutaneous fat, and moisture in her skin. d. Facial skin becomes more elastic with age. This increased elasticity causes the skin to be more taught, drawing attention to the facial bones.

c. More noticeable facial bones are probably due to a combination of factors related to aging, such as decreased elasticity, subcutaneous fat, and moisture in her skin. Rationale: The facial bones and orbits appear more prominent in the aging adult, and the facial skin sags, which is attributable to decreased elasticity, decreased subcutaneous fat, and decreased moisture in the skin.

A 60-year-old man is at the clinic for an eye examination. The nurse suspects that he has ptosis of one eye. How should the nurse check for this? a. Perform the confrontation test. b. Assess the individuals near vision. c. Observe the distance between the palpebral fissures. d. Perform the corneal light test, and look for symmetry of the light reflex.

c. Observe the distance between the palpebral fissures. Rationale: Ptosis is a drooping of the upper eyelid that would be apparent by observing the distance between the upper and lower eyelids. The confrontation test measures peripheral vision. Measuring near vision or the corneal light test does not check for ptosis.

During an examination, the nurse can assess mental status by which activity? a. Examining the patients electroencephalogram b. Observing the patient as he or she performs an intelligence quotient (IQ) test c. Observing the patient and inferring health or dysfunction d. Examining the patients response to a specific set of questions

c. Observing the patient and inferring health or dysfunction Rationale: Mental status cannot be directly scrutinized like the characteristics of skin or heart sounds. Its functioning is inferred through an assessment of an individuals behaviors, such as consciousness, language, mood and affect, and other aspects.

A 54-year-old man comes to the clinic with a horrible problem. He tells the nurse that he has just discovered a lump on his breast and is fearful of cancer. The nurse knows which statement about breast cancer in men is true? a. Breast masses in men are difficult to detect because of minimal breast tissue. b. Breast cancer in men rarely spreads to the lymph nodes. c. One percent of all breast cancers occurs in men. d. Most breast masses in men are diagnosed as gynecomastia.

c. One percent of all breast cancers occurs in men. Rationale: One percent of all breast cancers occurs in men. The early spreading to axillary lymph nodes is attributable to minimal breast tissue.

While examining a 75-year-old woman, the nurse notices that the skin over her right breast is thickened and the hair follicles are exaggerated. This condition is known as: a. Dimpling. b. Retraction. c. Peau d'orange. d. Benign breast disease.

c. Peau d'orange. Rationale: This condition is known as peau dorange. Lymphatic obstruction produces edema, which thickens the skin and exaggerates the hair follicles. The skin has a pig-skin or orange-peel appearance, and this condition suggests cancer.

The nurse is planning health teaching for a 65-year-old woman who has had a cerebrovascular accident (stroke) and has aphasia. Which of these questions is most important to use when assessing mental status in this patient? a. Please count backward from 100 by seven. b. I will name three items and ask you to repeat them in a few minutes. c. Please point to articles in the room and parts of the body as I name them. d. What would you do if you found a stamped, addressed envelope on the sidewalk?

c. Please point to articles in the room and parts of the body as I name them. Rationale: Additional tests for persons with aphasia include word comprehension (asking the individual to point to articles in the room or parts of the body), reading (asking the person to read available print), and writing (asking the person to make up and write a sentence).

The nurse is assessing an 80-year-old male patient. Which assessment findings would be considered normal? a. Increase in body weight from his younger years b. Additional deposits of fat on the thighs and lower legs c. Presence of kyphosis and flexion in the knees and hips d. Change in overall body proportion, including a longer trunk and shorter extremities

c. Presence of kyphosis and flexion in the knees and hips Rationale: Changes that occur in the aging person include more prominent bony landmarks, decreased body weight (especially in men), a decrease in subcutaneous fat from the face and periphery, and additional fat deposited on the abdomen and hips. Postural changes of kyphosis and slight flexion in the knees and hips also occur.

When a light is directed across the iris of a patients eye from the temporal side, the nurse is assessing for: a. Drainage from dacryocystitis. b. Presence of conjunctivitis over the iris. c. Presence of shadows, which may indicate glaucoma. d. Scattered light reflex, which may be indicative of cataracts.

c. Presence of shadows, which may indicate glaucoma. Rationale: The presence of shadows in the anterior chamber may be a sign of acute angle-closure glaucoma. The normal iris is flat and creates no shadows. This method is not correct for the assessment of dacryocystitis, conjunctivitis, or cataracts.

In a patient who has anisocoria, the nurse would expect to observe: a. Dilated pupils. b. Excessive tearing. c. Pupils of unequal size. d. Uneven curvature of the lens.

c. Pupils of unequal size. Rationale: Unequal pupil size is termed anisocoria. It normally exists in 5% of the population but may also be indicative of central nervous system disease.

The nurse has palpated a lump in a female patients right breast. The nurse documents this as a small, round, firm, distinct, lump located at 2 oclock, 2 cm from the nipple. It is nontender and fixed. No associated retraction of the skin or nipple, no erythema, and no axillary lymphadenopathy are observed. What information is missing from the documentation? a. Shape of the lump b. Consistency of the lump c. Size of the lump d. Whether the lump is solitary or multiple

c. Size of the lump Rationale: If the nurse feels a lump or mass, then he or she should note these characteristics: (1) location, (2) sizejudge in centimeters in three dimensions: width length thickness, (3) shape, (4) consistency, (5) motility, (6) distinctness, (7) nipple, (8) the skin over the lump, (9) tenderness, and (10) lymphadenopathy.

During an interview, the nurse would expect that most of the interview will take place at what distance? a. Intimate zone b. Personal distance c. Social distance d. Public distance

c. Social distance Rationale: Social distance, 4 to 12 feet, is usually the distance category for most of the interview. Public distance, over 12 feet, is too much distance; the intimate zone is inappropriate, and the personal distance will be used for the physical assessment.

The area of the nervous system that is responsible for mediating reflexes is the: a. Medulla. b. Cerebellum. c. Spinal cord. d. Cerebral cortex.

c. Spinal cord. Rationale: The spinal cord is the main highway for ascending and descending fiber tracts that connect the brain to the spinal nerves; it is responsible for mediating reflexes.

Which of these statements is true regarding the use of Standard Precautions in the health care setting? a. Standard Precautions apply to all body fluids, including sweat. b. Use alcohol-based hand rub if hands are visibly dirty. c. Standard Precautions are intended for use with all patients, regardless of their risk or presumed infection status. d. Standard Precautions are to be used only when nonintact skin, excretions containing visible blood, or expected contact with mucous membranes is present.

c. Standard Precautions are intended for use with all patients, regardless of their risk or presumed infection status. Rationale: Standard Precautions are designed to reduce the risk of transmission of microorganisms from both recognized and unrecognized sources and are intended for use for all patients, regardless of their risk or presumed infection status. Standard Precautions apply to blood and all other body fluids, secretions and excretions except sweat regardless of whether they contain visible blood, non-intact skin, or mucous membranes. Hands should be washed with soap and water if visibly soiled with blood or body fluids. Alcohol-based hand rubs can be used if hands are not visibly soiled.

A patient tells the nurse that he is very nervous, is nauseated, and feels hot. These types of data would be: a. Objective b. Reflective c. Subjective d. Introspective

c. Subjective Rationale: Subjective data are what the person says about him or herself during history taking. Objective data are what the health professional observes by inspecting, percussing, palpating, and auscultating during the physical examination. The terms reflective and introspective are not used to describe data.

During an examination, the nurse notices that a male patient has a red, round, superficial ulcer with a yellowish serous discharge on his penis. On palpation, the nurse finds a nontender base that feels like a small button between the thumb and fingers. At this point the nurse suspects that this patient has: a. Genital warts. b. Herpes infection. c. Syphilitic chancre. d. Carcinoma lesion.

c. Syphilitic chancre. Rationale: This lesion indicates syphilitic chancre, which begins within 2 to 4 weeks of infection. (See Table 24-4 for the descriptions of the other options.)

During an examination, the nurse finds that a patients left temporal artery is tortuous and feels hardened and tender, compared with the right temporal artery. The nurse suspects which condition? a. Crepitation b. Mastoiditis c. Temporal arteritis d. Bell palsy

c. Temporal arteritis Rationale: With temporal arteritis, the artery appears more tortuous and feels hardened and tender. These assessment findings are not consistent with the other responses.

When a breastfeeding mother is diagnosed with a breast abscess, which of these instructions from the nurse is correct? The mother needs to: a. Continue to nurse on both sides to encourage milk flow. b. Immediately discontinue nursing to allow for healing. c. Temporarily discontinue nursing on the affected breast, and manually express milk and discard it. d. Temporarily discontinue nursing on affected breast, but manually express milk and give it to the baby.

c. Temporarily discontinue nursing on the affected breast, and manually express milk and discard it. Rationale: With a breast abscess, the patient must temporarily discontinue nursing on the affected breast, manually express the milk, and then discard it. Nursing can continue on the unaffected side.

A patient has a severed spinal nerve as a result of trauma. Which statement is true in this situation? a. Because there are 31 pairs of spinal nerves, no effect results if only one nerve is severed. b. The dermatome served by this nerve will no longer experience any sensation. c. The adjacent spinal nerves will continue to carry sensations for the dermatome served by the severed nerve. d. A severed spinal nerve will only affect motor function of the patient because spinal nerves have no sensory component.

c. The adjacent spinal nerves will continue to carry sensations for the dermatome served by the severed nerve. Rationale: A dermatome is a circumscribed skin area that is primarily supplied from one spinal cord segment through a particular spinal nerve. The dermatomes overlap, which is a form of biologic insurance; that is, if one nerve is severed, then most of the sensations can be transmitted by the spinal nerve above and the spinal nerve below the severed nerve.

A patients blood pressure is 118/82 mm Hg. He asks the nurse, What do the numbers mean? The nurses best reply is: a. The numbers are within the normal range and are nothing to worry about. b. The bottom number is the diastolic pressure and reflects the stroke volume of the heart. c. The top number is the systolic blood pressure and reflects the pressure of the blood against the arteries when the heart contracts. d. The concept of blood pressure is difficult to understand. The primary thing to be concerned about is the top number, or the systolic blood pressure.

c. The top number is the systolic blood pressure and reflects the pressure of the blood against the arteries when the heart contracts. Rationale: The systolic pressure is the maximum pressure felt on the artery during left ventricular contraction, or systole. The diastolic pressure is the elastic recoil, or resting, pressure that the blood constantly exerts in between each contraction. The nurse should answer the patients question and use terms he can understand.

The nurse is unable to palpate the right radial pulse on a patient. The best action would be to: a. Auscultate over the area with a fetoscope. b. Use a goniometer to measure the pulsations. c. Use a Doppler device to check for pulsations over the area. d. Check for the presence of pulsations with a stethoscope.

c. Use a Doppler device to check for pulsations over the area. Rationale: Doppler devices are used to augment pulse or blood pressure measurements. Goniometers measure joint range of motion. A fetoscope is used to auscultate fetal heart tones. Stethoscopes are used to auscultate breath, bowel, and heart sounds.

The nurse is preparing to assess the visual acuity of a 16-year-old patient. How should the nurse proceed? a. Perform the confrontation test. b. Ask the patient to read the print on a handheld Jaeger card. c. Use the Snellen chart positioned 20 feet away from the patient. d. Determine the patients ability to read newsprint at a distance of 12 to 14 inches.

c. Use the Snellen chart positioned 20 feet away from the patient. Rationale: The Snellen alphabet chart is the most commonly used and most accurate measure of visual acuity. The confrontation test is a gross measure of peripheral vision. The Jaeger card or newspaper tests are used to test near vision.

The nurse notices that a patients palpebral fissures are not symmetric. On examination, the nurse may find that damage has occurred to which cranial nerve (CN)? a. III b. V c. VII d. VIII

c. VII Rationale: Facial muscles are mediated by CN VII; asymmetry of palpebral fissures may be attributable to damage to CN VII (Bell palsy).

When listening to a patients breath sounds, the nurse is unsure of a sound that is heard. The nurses next action should be to: a. Immediately notify the patients physician b. Document the sound exactly as it was heard c. Validate the data by asking a coworker to listen to the breath sounds d. Assess again in 20 minutes to note whether the sound is still present

c. Validate the data by asking a coworker to listen to the breath sounds Rationale: When unsure of a sound heard while listening to a patients breath sounds, the nurse validates the data to ensure accuracy. If the nurse has less experience in an area, then he or she asks an expert to listen.

The most important step that the nurse can take to prevent the transmission of microorganisms in the hospital setting is to: a. Wear protective eye wear at all times. b. Wear gloves during any and all contact with patients. c. Wash hands before and after contact with each patient. d. Clean the stethoscope with an alcohol swab between patients.

c. Wash hands before and after contact with each patient. Rationale: The most important step to decrease the risk of microorganism transmission is to wash hands promptly and thoroughly before and after physical contact with each patient. Stethoscopes should also be cleansed with an alcohol swab before and after each patient contact. The best routine is to combine stethoscope rubbing with hand hygiene each time hand hygiene is performed.

During a physical examination, a 45-year-old woman states that she has had a crusty, itchy rash on her breast for approximately 2 weeks. In trying to find the cause of the rash, which question would be important for the nurse to ask? a. Is the rash raised and red? b. Does it appear to be cyclic? c. Where did the rash first appear, on the nipple, the areola, or the surrounding skin? d. What was she doing when she first noticed the rash, and do her actions make it worse?

c. Where did the rash first appear, on the nipple, the areola, or the surrounding skin? Rationale: The location where the rash first appeared is important for the nurse to determine. Paget disease starts with a small crust on the nipple apex and then spreads to the areola. Eczema or other dermatitis rarely starts at the nipple unless it is a result of breastfeeding. It usually starts on the areola or surrounding skin and then spreads to the nipple (see Table 17-6).

A patient has been in the intensive care unit for 10 days. He has just been moved to the medical-surgical unit, and the admitting nurse is planning to perform a mental status examination. During the tests of cognitive function, the nurse would expect that he: a. May display some disruption in thought content. b. Will state, I am so relieved to be out of intensive care. c. Will be oriented to place and person, but the patient may not be certain of the date. d. May show evidence of some clouding of his level of consciousness.

c. Will be oriented to place and person, but the patient may not be certain of the date. Rationale: The nurse can discern the orientation of cognitive function through the course of the interview or can directly and tactfully ask, Some people have trouble keeping up with the dates while in the hospital. Do you know todays date? Many hospitalized people have trouble with the exact date but are fully oriented on the remaining items.

Which of these correctly describes the average length of pregnancy? a. 38 weeks b. 9 lunar months c. 280 days from the last day of the last menstrual period d. 280 days from the first day of the last menstrual period

d. 280 days from the first day of the last menstrual period Rationale: The average length of pregnancy is 280 days from the first day of the last menstrual period, which is equal to 40 weeks, 10 lunar months, or roughly 9 calendar months.

The nurse is reviewing causes of increased intraocular pressure. Which of these factors determines intraocular pressure? a. Thickness or bulging of the lens b. Posterior chamber as it accommodates increased fluid c. Contraction of the ciliary body in response to the aqueous within the eye d. Amount of aqueous produced and resistance to its outflow at the angle of the anterior chamber

d. Amount of aqueous produced and resistance to its outflow at the angle of the anterior chamber Rationale: Intraocular pressure is determined by a balance between the amount of aqueous produced and the resistance to its outflow at the angle of the anterior chamber. The other responses are incorrect.

A woman who is 28 weeks pregnant has bilateral edema in her lower legs after working 8 hours a day as a cashier at a local grocery store. She is worried about her legs. What is the nurses best response? a. You will be at risk for development of varicose veins when your legs are edematous. b. I would like to listen to your heart sounds. Edema can indicate a problem with your heart. c. Edema is usually the result of too much salt and fluids in your diet. You may need to cut down on salty foods. d. As your baby grows, it slows blood return from your legs, causing the swelling. This often occurs with prolonged standing.

d. As your baby grows, it slows blood return from your legs, causing the swelling. This often occurs with prolonged standing. Rationale: Edema of the lower extremities occurs because of the enlarging fetus, which impairs venous return. Prolonged standing worsens the edema. Typically, the bilateral, dependent edema experienced with pregnancy is not the result of a cardiac pathologic condition.

The nursing process is a sequential method of problem solving that nurses use and includes which steps? a. Assessment, treatment, planning, evaluation, discharge, and follow-up b. Admission, assessment, diagnosis, treatment, and discharge planning c. Admission, diagnosis, treatment, evaluation, and discharge planning d. Assessment, diagnosis, outcome identification, planning, implementation, and evaluation

d. Assessment, diagnosis, outcome identification, planning, implementation, and evaluation Rationale: The nursing process is a method of problem solving that includes assessment, diagnosis, outcome identification, planning, implementation, and evaluation.

A 50-year-old woman is in the clinic for weakness in her left arm and leg that she has noticed for the past week. The nurse should perform which type of neurologic examination? a. Glasgow Coma Scale b. Neurologic recheck examination c. Screening neurologic examination d. Complete neurologic examination

d. Complete neurologic examination Rationale: The nurse should perform a complete neurologic examination on an individual who has neurologic concerns (e.g., headache, weakness, loss of coordination) or who is showing signs of neurologic dysfunction. The Glasgow Coma Scale is used to define a persons level of consciousness. The neurologic recheck examination is appropriate for those who are demonstrating neurologic deficits. The screening neurologic examination is performed on seemingly well individuals who have no significant subjective findings from the health history.

The nurse is assessing a patients eyes for the accommodation response and would expect to see which normal finding? a. Dilation of the pupils b. Consensual light reflex c. Conjugate movement of the eyes d. Convergence of the axes of the eyes

d. Convergence of the axes of the eyes Rationale: The accommodation reaction includes pupillary constriction and convergence of the axes of the eyes. The other responses are not correct.

When the nurse is performing a genital examination on a male patient, which action is correct? a. Auscultating for the presence of a bruit over the scrotum b. Palpating for the vertical chain of lymph nodes along the groin, inferior to the inguinal ligament c. Palpating the inguinal canal only if a bulge is present in the inguinal region during inspection d. Having the patient shift his weight onto the left (unexamined) leg when palpating for a hernia on the right side

d. Having the patient shift his weight onto the left (unexamined) leg when palpating for a hernia on the right side Rationale: When palpating for the presence of a hernia on the right side, the male patient is asked to shift his weight onto the left (unexamined) leg. Auscultating for a bruit over the scrotum is not appropriate. When palpating for lymph nodes, the horizontal chain is palpated. The inguinal canal should be palpated whether a bulge is present or not.

A patient has been diagnosed with schizophrenia. During a recent interview, he shows the nurse a picture of a man holding a decapitated head. He describes this picture as horrifying but then laughs loudly at the content. This behavior is a display of: a. Confusion b. Ambivalence c. Depersonalization d. Inappropriate affect

d. Inappropriate affect Rationale: An inappropriate affect is an affect clearly discordant with the content of the persons speech. (See Table 5-5 for the definitions of the other terms.)

A 20-year-old construction worker has been brought into the emergency department with heat stroke. He has delirium as a result of a fluid and electrolyte imbalance. For the mental status examination, the nurse should first assess the patients: a. Affect and mood b. Memory and affect c. Language abilities d. Level of consciousness and cognitive abilities

d. Level of consciousness and cognitive abilities Rationale: Delirium is a disturbance of consciousness (i.e., reduced clarity of awareness of the environment) with reduced ability to focus, sustain, or shift attention. Delirium is not an alteration in mood, affect, or language abilities.

The nurse is testing the function of CN XI. Which statement best describes the response the nurse should expect if this nerve is intact? The patient: a. Demonstrates the ability to hear normal conversation. b. Sticks out the tongue midline without tremors or deviation. c. Follows an object with his or her eyes without nystagmus or strabismus. d. Moves the head and shoulders against resistance with equal strength.

d. Moves the head and shoulders against resistance with equal strength. Rationale: The following normal findings are expected when testing the spinal accessory nerve (CN XI): The patients sternomastoid and trapezius muscles are equal in size; the person can forcibly rotate the head both ways against resistance applied to the side of the chin with equal strength; and the patient can shrug the shoulders against resistance with equal strength on both sides. Checking the patients ability to hear normal conversation checks the function of CN VIII. Having the patient stick out the tongue checks the function of CN XII. Testing the eyes for nystagmus or strabismus is performed to check CNs III, IV, and VI.

When the nurse asks a 68-year-old patient to stand with his feet together and arms at his side with his eyes closed, he starts to sway and moves his feet farther apart. The nurse would document this finding as: a. Ataxia. b. Lack of coordination. c. Negative Homans sign. d. Positive Romberg sign.

d. Positive Romberg sign. Rationale: Abnormal findings for the Romberg test include swaying, falling, and a widening base of the feet to avoid falling. A positive Romberg sign is a loss of balance that is increased by the closing of the eyes. Ataxia is an uncoordinated or unsteady gait. Homans sign is used to test the legs for deep-vein thrombosis.

The nurse is palpating the abdomen of a woman who is 35 weeks pregnant and notices that the fetal head is facing downward toward the pelvis. The nurse would document this as fetal: a. Lie. b. Variety. c. Attitude. d. Presentation.

d. Presentation. Rationale: Fetal presentation describes the part of the fetus that is entering the pelvis first. Fetal lie is orientation of the fetal spine to the maternal spine. Fetal attitude is the position of fetal parts in relation to each other, and fetal variety is the location of the fetal back to the maternal pelvis.

A female patient has nausea, breast tenderness, fatigue, and amenorrhea. Her last menstrual period was 6 weeks ago. The nurse interprets that this patient is experiencing __________ signs of pregnancy. a. Positive b. Possible c. Probable d. Presumptive

d. Presumptive Rationale: Presumptive signs of pregnancy are those that the woman experiences and include amenorrhea, breast tenderness, fatigue, nausea, and increased urinary frequency. Probable signs are those that are detected by the examiner, such as an enlarged uterus or changes in the cervix. Positive signs of pregnancy are those that document direct evidence of the fetus such as fetal heart tones or positive cardiac activity on ultrasound.

Which of these statements best describes the action of the hormone progesterone during pregnancy? a. Progesterone produces the hormone human chorionic gonadotropin. b. Duct formation in the breast is stimulated by progesterone. c. Progesterone promotes sloughing of the endometrial wall. d. Progesterone maintains the endometrium around the fetus.

d. Progesterone maintains the endometrium around the fetus. Rationale: Progesterone prevents the sloughing of the endometrial wall and maintains the endometrium around the fetus. Progesterone increases the alveoli in the breast and keeps the uterus in a quiescent state. The other options are not correct.

The nurse is assessing an older adults functional ability. Which definition correctly describes ones functional ability? Functional ability: a. Is the measure of the expected changes of aging that one is experiencing. b. Refers to the individuals motivation to live independently. c. Refers to the level of cognition present in an older person. d. Refers to ones ability to perform activities necessary to live in modern society.

d. Refers to ones ability to perform activities necessary to live in modern society. Rationale: Functional ability refers to ones ability to perform activities necessary to live in modern society and can include driving, using the telephone, or performing personal tasks such as bathing and toileting.

A patient comes into the emergency department after an accident at work. A machine blew dust into his eyes, and he was not wearing safety glasses. The nurse examines his corneas by shining a light from the side across the cornea. What findings would suggest that he has suffered a corneal abrasion? a. Smooth and clear corneas b. Opacity of the lens behind the cornea c. Bleeding from the areas across the cornea d. Shattered look to the light rays reflecting off the cornea

d. Shattered look to the light rays reflecting off the cornea Rationale: A corneal abrasion causes irregular ridges in reflected light, which produce a shattered appearance to light rays. No opacities should be observed in the cornea. The other responses are not correct.

The nurse is examining the glans and knows which finding is normal for this area? a. The meatus may have a slight discharge when the glans is compressed. b. Hair is without pest inhabitants. c. The skin is wrinkled and without lesions. d. Smegma may be present under the foreskin of an uncircumcised male.

d. Smegma may be present under the foreskin of an uncircumcised male. Rationale: The glans looks smooth and without lesions and does not have hair. The meatus should not have any discharge when the glans is compressed. Some cheesy smegma may have collected under the foreskin of an uncircumcised male.

During ocular examinations, the nurse keeps in mind that movement of the extraocular muscles is: a. Decreased in the older adult. b. Impaired in a patient with cataracts. c. Stimulated by cranial nerves (CNs) I and II. d. Stimulated by CNs III, IV, and VI.

d. Stimulated by CNs III, IV, and VI. Rationale: Movement of the extraocular muscles is stimulated by three CNs: III, IV, and VI.

During the interview portion of data collection, the nurse collects __________ data. a. Physical b. Historical c. Objective d. Subjective

d. Subjective Rationale: The interview is the first, and really the most important, part of data collection. During the interview, the nurse collects subjective data; that is, what the person says about him or herself.

During an examination of a 3-year-old child, the nurse will need to take her blood pressure. What might the nurse do to try to gain the child's full cooperation? a. Tell the child that the blood pressure cuff is going to give her arm a big hug. b. Tell the child that the blood pressure cuff is asleep and cannot wake up. c. Give the blood pressure cuff a name and refer to it by this name during the assessment. d. Tell the child that by using the blood pressure cuff, we can see how strong her muscles are.

d. Tell the child that by using the blood pressure cuff, we can see how strong her muscles are. Rationale: Take the time to give a short, simple explanation with a concrete explanation for any unfamiliar equipment that will be used on the child. Preschoolers are animistic; they imagine inanimate objects can come alive and have human characteristics. Thus a blood pressure cuff can wake up and bite or pinch.

The nurse is preparing to assess a patients abdomen by palpation. How should the nurse proceed? a. Palpation of reportedly tender areas are avoided because palpation in these areas may cause pain. b. Palpating a tender area is quickly performed to avoid any discomfort that the patient may experience. c. The assessment begins with deep palpation, while encouraging the patient to relax and to take deep breaths. d. The assessment begins with light palpation to detect surface characteristics and to accustom the patient to being touched.

d. The assessment begins with light palpation to detect surface characteristics and to accustom the patient to being touched. Rationale: Light palpation is initially performed to detect any surface characteristics and to accustom the person to being touched. Tender areas should be palpated last, not first.

Which of these statements is true regarding the penis? a. The urethral meatus is located on the ventral side of the penis. b. The prepuce is the fold of foreskin covering the shaft of the penis. c. The penis is made up of two cylindrical columns of erectile tissue. d. The corpus spongiosum expands into a cone of erectile tissue called the glans.

d. The corpus spongiosum expands into a cone of erectile tissue called the glans. Rationale: At the distal end of the shaft, the corpus spongiosum expands into a cone of erectile tissue, the glans. The penis is made up of three cylindrical columns of erectile tissue. The skin that covers the glans of the penis is the prepuce. The urethral meatus forms at the tip of the glans.

A 65-year-old patient remarks that she just cannot believe that her breasts sag so much. She states it must be from a lack of exercise. What explanation should the nurse offer her? After menopause: a. Only women with large breasts experience sagging. b. Sagging is usually due to decreased muscle mass within the breast. c. A diet that is high in protein will help maintain muscle mass, which keeps the breasts from sagging. d. The glandular and fat tissue atrophies, causing breast size and elasticity to diminish, resulting in breasts that sag.

d. The glandular and fat tissue atrophies, causing breast size and elasticity to diminish, resulting in breasts that sag. Rationale: After menopause, the glandular tissue atrophies and is replaced with connective tissue. The fat envelope also atrophies, beginning in the middle years and becoming significant in the eighth and ninth decades of life. These changes decrease breast size and elasticity; consequently, the breasts droop and sag, looking flattened and flabby.

During an assessment, the nurse uses the CAGE test. The patient answers yes to two of the questions. What could this be indicating? a. The patient is an alcoholic. b. The patient is annoyed at the questions. c. The patient should be thoroughly examined for possible alcohol withdrawal symptoms. d. The nurse should suspect alcohol abuse and continue with a more thorough substance abuse assessment.

d. The nurse should suspect alcohol abuse and continue with a more thorough substance abuse assessment. Rationale: The CAGE test is known as the cut down, annoyed, guilty, and eye-opener test. If a person answers yes to two or more of the four CAGE questions, then the nurse should suspect alcohol abuse and continue with a more complete substance abuse assessment.

Which of these statements about the peripheral nervous system is correct? a. The CNs enter the brain through the spinal cord. b. Efferent fibers carry sensory input to the central nervous system through the spinal cord. c. The peripheral nerves are inside the central nervous system and carry impulses through their motor fibers. d. The peripheral nerves carry input to the central nervous system by afferent fibers and away from the central nervous system by efferent fibers.

d. The peripheral nerves carry input to the central nervous system by afferent fibers and away from the central nervous system by efferent fibers. Rationale: A nerve is a bundle of fibers outside of the central nervous system. The peripheral nerves carry input to the central nervous system by their sensory afferent fibers and deliver output from the central nervous system by their efferent fibers. The other responses are not related to the peripheral nervous system.

A patient is describing his symptoms to the nurse. Which of these statements reflects a description of the setting of his symptoms? a. It is a sharp, burning pain in my stomach. b. I also have the sweats and nausea when I feel this pain. c. I think this pain is telling me that something bad is wrong with me. d. This pain happens every time I sit down to use the computer.

d. This pain happens every time I sit down to use the computer. Rationale: The setting describes where the person is or what the person is doing when the symptom starts. Describing the pain as sharp and burning reflects the character or quality of the pain; stating that the pain is telling the patient that something bad is wrong with him reflects the patients perception of the pain; and describing the sweats and nausea reflects associated factors that occur with the pain.

The nurse is preparing to conduct a health history. Which of these statements best describes the purpose of a health history? a. To provide an opportunity for interaction between the patient and the nurse b. To provide a form for obtaining the patients biographic information c. To document the normal and abnormal findings of a physical assessment d. To provide a database of subjective information about the patients past and current health

d. To provide a database of subjective information about the patients past and current health Rationale: The purpose of the health history is to collect subjective data, which is what the person says about him or herself. The other options are not correct.

During a womans 34th week of pregnancy, she is told that she has preeclampsia. The nurse knows which statement concerning preeclampsia is true? a. Preeclampsia has little effect on the fetus. b. Edema is one of the main indications of preeclampsia. c. Eclampsia only occurs before delivery of the baby. d. Untreated preeclampsia may contribute to restriction of fetal growth.

d. Untreated preeclampsia may contribute to restriction of fetal growth. Rationale: Untreated preeclampsia may progress to eclampsia, which is manifested by generalized tonic-clonic seizures. Eclampsia may develop as late as 10 days postpartum. Before the syndrome becomes clinically manifested, it is affecting the placenta through vasospasm and a series of small infarctions. The placentas capacity to deliver oxygen and nutrients may be seriously diminished, and fetal growth may be restricted. Edema is common in pregnancy and is not an indicator of preeclampsia.

A 15-year-old boy is seen in the clinic for complaints of dull pain and pulling in the scrotal area. On examination, the nurse palpates a soft, irregular mass posterior to and above the testis on the left. This mass collapses when the patient is supine and refills when he is upright. This description is consistent with: a. Epididymitis. b. Spermatocele. c. Testicular torsion. d. Varicocele.

d. Varicocele. Rationale:A varicocele consists of dilated, tortuous varicose veins in the spermatic cord caused by incompetent valves within the vein. Symptoms include dull pain or a constant pulling or dragging feeling, or the individual may be asymptomatic. When palpating the mass, the examiner will feel a soft, irregular mass posterior to and above the testis that collapses when the individual is supine and refills when the individual is upright. (See Table 24-6 for more information and for the descriptions of the other options.)

A patient has had a cerebrovascular accident (stroke). He is trying very hard to communicate. He seems driven to speak and says, I buy obie get spirding and take my train. What is the best description of this patients problem? a. Global aphasia b. Brocas aphasia c. Echolalia d. Wernicke's aphasia

d. Wernicke's aphasia Rationale: This type of communication illustrates Wernicke's or receptive aphasia. The person can hear sounds and words but cannot relate them to previous experiences. Speech is fluent, effortless, and well articulated, but it has many paraphasias (word substitutions that are malformed or wrong) and neologisms (made-up words) and often lacks substantive words. Speech can be totally incomprehensible. Often, a great urge to speak is present. Repetition, reading, and writing also are impaired. Echolalia is an imitation or the repetition of another persons words or phrases. (See Table 5-4 for the definitions of the other disorders.)

The nurse is teaching a pregnant woman about breast milk. Which statement by the nurse is correct? a. Your breast milk is immediately present after the delivery of your baby. b. Breast milk is rich in protein and sugars (lactose) but has very little fat. c. The colostrum, which is present right after birth, does not contain the same nutrients as breast milk. d. You may notice a thick, yellow fluid expressed from your breasts as early as the fourth month of pregnancy.

d. You may notice a thick, yellow fluid expressed from your breasts as early as the fourth month of pregnancy. Rationale: After the fourth month, colostrum may be expressed. This thick yellow fluid is the precursor of milk, and it contains the same amount of protein and lactose but practically no fat. The breasts produce colostrum for the first few days after delivery. It is rich with antibodies that protect the newborn against infection; therefore, breastfeeding is important.

A 70-year-old woman tells the nurse that every time she gets up in the morning or after shes been sitting, she gets really dizzy and feels like she is going to fall over. The nurses best response would be: a. Have you been extremely tired lately? b. You probably just need to drink more liquids. c. I'll refer you for a complete neurologic examination. d. You need to get up slowly when you've been lying down or sitting.

d. You need to get up slowly when you've been lying down or sitting. Rationale: Aging is accompanied by a progressive decrease in cerebral blood flow. In some people, this decrease causes dizziness and a loss of balance with a position change. These individuals need to be taught to get up slowly. The other responses are incorrect.


संबंधित स्टडी सेट्स

Engl305 Shakespeare and Contemporaries

View Set

6.02 Health: Injuries and Taking Risks

View Set

Science- lesson 1 & 2- Sun- Earth- Moon// Solar System

View Set

Biomechanics Final Exam Review: Projectile Motion

View Set

Elections and Political Participation Vocabulary

View Set

ATI Engage Fundamentals PN foundational concepts of nursing practice patient centered care

View Set

Hematology Test Review Questions

View Set

Social Studies Ch.2 lesson 1-3 ;)

View Set

Daddy-Positive Organizational Behavior and Psychological Capital- Ch 7

View Set